ORTHOPEDIC MCQS ONLINE OB SHOULDER AND ELBOW 1B
ORTHOPEDIC MCQS ONLINE OB SHOULDER AND ELBOW 1B
-
Which of the following rehabilitation exercises is most appropriate immediately following the repair of the injury seen in figure A?
Passive external rotation at 90 degrees of abduction
Isotonic rotator cuff strengthening Isokinetic resistive elbow flexion
Passive and active assisted flexion in scapular plane Concentric latissimus pull down exercises
CORRECT ANSWER: 4
Figure A shows a superior labral anterior to posterior (SLAP) tear. Passive and active-assisted flexion in scapular plane are usually allowed immediately postoperatively. In general, the early focus is on healing and re-establishing motion, followed by strength training. Post-operative protocols do vary among surgeons (some delay active assisted range of motion) as there is no high level evidence supporting a certain protocol.
SLAP tears can occur as isolated lesions or associated with other injuries such as: internal impingement commonly seen in overhead throwers, rotator cuff tears (usually partial-articular sided), and instability (including microinstability).
In a biomechanical cadaver study, Shepard et al. tested if the direction of biceps anchor loading would result in differences in the ultimate strength of the biceps anchor and the generation of SLAP lesions. They found that the biceps anchor was significantly weaker when loaded with a posterior vector, as opposed to an in-line pull, and concluded that the superior labrum may be most vulnerable to injury in late cocking.
Figure A shows a T2 coronal MRI that demonstrates an isolated SLAP tear. Incorrect answers:
Answer 1- Would place stress on a SLAP repair, and is usually avoided for about 4 weeks. Answer 2- Isotonic shoulder strengthening exercises are usually initiated around weeks 4-6 (isometrics for muscle activation may be started earlier).
Answer 3- Resisted active isokinetic elbow flexion would place stress on a SLAP repair, and is generally usually avoided for 6 weeks.
Answer 5- Concentric latissimus pull down exercises would generally be avoided as immediate shoulder abduction and active elbow flexion would place stress at the SLAP repair site.
-
A 72-year-old male presents with left shoulder and scapular pain after a fall 2 months ago. A clinical picture is shown in Figure A. Which of the following is most likely affected?
Nerve roots C 4-7 Nerve roots C 6-7 Cranial nerve XI Nerve roots C 3-5 Cranial nerve XII
CORRECT ANSWER: 3
The clinical photo is consistent with lateral scapular winging, which is caused by damage to the spinal accessory nerve, or cranial nerve XI.
Lateral scapular winging is a rare condition caused by trauma to cranial nerve XI, or the spinal accessory nerve. The spinal accessory nerve has a superficial course in the posterior triangle of the neck, placing it at risk during traumatic injuries to the arm or neck. EMG studies are an important tool for appropriate diagnosis, and are often repeated every three to six months to test for improvement. Initial treatment is non-operative, with procedure such as the Eden-Lange transfer or scapulothoracic fusion being reserved for refractory cases.
Shin et al. discuss traumatic brachial plexus injuries in adults. They urge surgery to be considered in the absence of clinical or electrical evidence of recovery or when spontaneous recovery is impossible.
Meininger et al. review scapular winging. They discuss that medial scapular winging caused by damage to the long thoracic nerve is the most common cause of primary scapular winging. With regards to lateral scapular winging, the most common cause is iatrogenic injury to the spinal accessory nerve during cervical lymph node biopsy or mass excision.
Figure A shows an example of lateral scapular winging caused by damage to the spinal accessory nerve, or cranial nerve XI. There is lateral displacement of the scapula with the superior angle more lateral to the midline than the inferior angle. In contrast, Illustration A shows medial scapular winging, caused by damage to the long thoracic nerve. Notice the medial border lifting off the posterior thoracic wall along with the medial and superior translation of the right scapula when compared to the normal left side.
Incorrect Answers:
Answers 1, 2, 4, 5: Damage to these nerves will not cause lateral scapular winging.
-
A 45-year-old patient presents with pain and swelling after undergoing an arthroscopic rotator cuff repair 10 weeks ago. On physical exam the portal sites are healed and there is no drainage. Testing of the integrity of the rotator cuff is limited secondary to pain. He has a WBC of 11.0 (reference range, 3-11 cells/mL), ESR of 40 mm/hr (reference range, 0-22 mm/hr), and CRP of 1.5 mg/dL ( reference range, 0-1 mg/dL). An aspiration is completed and no organisms are seen on the gram stain. Twelve days after the aspiration, positive cultures are reported. Which organism is most likely to have grown in culture medium?
1 . Staphylococcus aureus 2 . Propionibacterium acnes Corynebacterium sp.
Staphylococcus epidermidis Pseudomonas aeruginosa
CORRECT ANSWER: 2
Based on the delayed clinical presentation and laboratory findings, the most likely bacterium to have grown in culture is Propionibacterium acnes (P. acnes).
Infections after arthroscopic rotator cuff repairs have a reported prevalence of
0.006%-3.4 %. The typical skin flora includes staph and strep as well as P. acnes, which has a propensity for the shoulder. Because it is an anaerobic organism, cultures may only become positive after 7-21 days.
Millett et al. reviewed 10 patients with post-operative shoulder infections and confirmed P. acnes infection. Cultures became positive at an average of 7 days after the start of incubation. They indicate that P. acnes shoulder infections may be insidious in onset and may lead to persistent shoulder pain.
Dodson et al. reviewed 11 patients who sustained P. acnes shoulder infections after undergoing shoulder arthroplasty. They found that cultures became positive at an average of 9 days from the start of incubation. They note that P. acnes shoulder infections are an important cause of implant failure, especially because patents may lack traditional signs of infection (clinical/laboratory).
Saltzman et al. review the microbiology, risk factors and management of infections after shoulder surgery. They indicate that deeper infections are more difficult to identify and can present in an insidious fashion. Cultures should be held for up to 21 days in order to account for fastidious organisms such as P. acnes., for 4 weeks to account for fungal organisms, and for 8 weeks to account for mycobacterium.
Matsen et al. studied the presence of P. acnes on skin and in surgical wounds of patients undergoing revision shoulder arthroplasty. Males were found to have more positive cultures for P.acnes than females; this included superficial and deep cultures. They recommend observing cultures for at least 17 days and being vigilant to minimize potential surgical wound contamination. Illustration A shows a gram stain of P. acnes, a gram-positive rod. Illustration B shows a scanning electron microscopic view of P. acnes.
Incorrect Answers:
Answers 1, 3, 4: While these bacteria are found on the skin are cause postoperative infections, they are less likely to present with an indolent course and prolonged incubation times.
Answer 5: Pseudomonas is a water-borne gram-negative rod that is not typically implicated in infections after shoulder surgery.
-
A 47-year-old, healthy, active patient presents with a sub-acute, full-thickness supraspinatus tear. His physical examination reveals significant weakness and pain with abduction. There was no glenohumeral instability. Radiographs demonstrate a type 1 acromion. An MRI scan shows a crescent shaped tear with 2-cm of tendinous retraction and no tendinous fatty changes. A subacromial corticosteroid injection 6 weeks ago provided him with 24 hours of pain relief but no improvement in strength. What would be the most appropriate treatment option?
Repeat subacromial corticosteriod injection
Biological augmentation of rotator cuff with porcine small intestine xenograft Rotator cuff repair
Rotator cuff repair plus acromioplasty
Rotator cuff repair, remplissage procedure, bicep tenodesis and distal clavicle excision
CORRECT ANSWER: 3
This patient has an isolated supraspinatus rotator cuff tear with symptomatic weakness. The most appropriate treatment would be isolated rotator cuff repair.
The primary purpose of rotator cuff repair is to restore muscle function. Secondary outcomes include reduction of pain and prevention of irreversible cuff changes, specifically muscular atrophy. Non-operative treatment ( exercise, therapy and pain medications) are recommended for partial thickness tears. The indication of surgical repair includes, isolated supraspinatus weakness +/- pain
that correlates with MRI imaging of a respective full thickness tear. Routine acrominoplasty is not recommended in conjunction with rotator cuff repair, especially with no previous symptoms of impingement.
Pedowitz et al. developed clinical practice guidelines for the treatment of rotator cuff pathology. The strongest supporting evidence in current literature was given a grade of 'moderate' with four treatment recommendations. These were,
Exercise and non-steroidal anti-inflammatory drugs can be used to manage partial thickness tears,
Routine acromioplasty is not required the time of cuff repair,
Non-cross-linked, porcine small intestine submucosal xenograft patches should not be used to manage cuff tears, and
Surgeons can advise patients that workers' compensation status correlates with a less favorable outcome after rotator cuff surgery.
Illustration A shows the different shapes of rotator cuff tears. Incorrect Answers:
Answer 1- Repeat subacromial corticosteriod injection could be considered, but this would not address this patients weakness.
Answer 2- There was moderate evidence against the use xenografts for full thickness rotator cuff repairs.
Answer 4- There was moderate evidence AGAINST routine acromioplasty in junction with rotator cuff repair, especially with no previous symptoms of impingement.
Answer 5- The remplissage procedure, bicep tenodesis and distal clavicle excision would not be warranted in this patient, as there is no supporting features in the clinical exam or imaging to support any additional pathology in the shoulder.
-
A 12-year-old baseball pitcher describes progressive worsening of medial elbow pain on
his throwing side. Examination reveals normal elbow range of motion. He is tender over the medial elbow to palpation. A dynamic ultrasound of his elbow shows no evidence of medial widening with valgus stress. His radiograph is shown in Figure A and an MRI is shown in Figure B. What is the most likely cause of his symptoms?
Displaced medial epicondyle avulsion fracture Medial apophysitis
Medial ulnar collateral ligament tear
Valgus extension overload with olecranon osteophytes Ulnar neuritis
CORRECT ANSWER: 2
The clinical presentation is consistent with Little League Elbow caused by medial apophysitis. Little League elbow is a general term explaining medial elbow pain in adolescent pitchers. The underlying pathology can include medial epicondyle stress fractures, avulsion fractures of the medial epicondyle, ulnar collateral ligament (UCL) injuries, or medial epicondyle apophysitis. In order to identify the underlying cause it is important to first rule out injury to the MCL by looking for medial widening on stress radiographs or dynamic ultrasound, or valgus instability on physical exam. Radiographs are useful to look for avulsion fractures or subtle physeal widening commonly seen with apophysitis.
Wei et al. obtained radiographs and magnetic resonance imaging on nine adolescent pitchers with a clinical diagnosis of Little League Elbow. They found radiographic findings in 4/9 and MRI findings in 6/9 patients. They emphasized that the MRI did not change management in any patients. Cain et al. review the different elbow conditions seen in throwing athletes. They emphasize the need to understand the underlying pathophysiology in order to treat and make appropriate changes to the biomechanics of the pitching technique.
Figure A shows an AP radiograph with slight widening of the apophysis, but no evidence of avulsion fracture. Figure B is an MRI which shows signal consistent with edema of the medial epicondyle apophysis.
Incorrect Answers:
The other responses are all typical throwing elbow conditions, but are much less common than apophysitis in the adolescent thrower.
-
What is the primary function of the structure labeled with an asterisk in Figure A?
Prevents inferior translation of the humerus with the arm by the side Provides internal rotation of the humerus
Prevents anterior translation of the humerus with the arm in 45 degrees of abduction Prevents anterior translation of the humerus with the arm in 90 degrees of abduction Provides supination of the forearm and elbow flexion
CORRECT ANSWER: 3
The labeled structure is the middle glenohumeral ligament (MGHL) of the shoulder. The primary function of the MGHL is to prevent anterior translation of the humeral head with the arm in 45-60 degrees of abduction.
This structure originates from the glenoid labrum and inserts medial to the lesser tuberosity running obliquely across the subscapularis. The size of the structure may be variable and there are recognized normal anatomic variants ( including a cord like MGHL in the Buford complex). It is important to be able to recognize the MGHL and differentiate this from the subscapularis, IGHL, SGHL, and other intraarticular structures in the shoulder to be able to perform effective and precise arthroscopic procedures.
Burkhart et al. describe the function of the glenohumeral ligaments in anterior shoulder instability, noting that the MGHL provides a restraint to anterior translation with the arm in 45-60 degrees of abduction.
Wang et al. discuss microdamage to the inferior glenohumeral ligament from a basic science perspective, indicating that over time it may stretch and compromise it's function in restraining humeral translation.
Figure A is an arthroscopic image of the intraarticular structures of the shoulder with an asterisk on the MGHL.
Incorrect Answers (these are labeled on Illustration A, with the exception of the subscapularis which is difficult to visualize):
Answer 1: Describes the superior glenohumeral ligament (1: Illustration A) Answer 2: Describes the subscapularis (2: Illustration A)
Answer 4: Describes the inferior glenohumeral ligament (4: Illustration A) Answer 5: Describes the biceps tendon (5: Illustration A)
-
In which of the following clinical circumstances would it be appropriate to eccentrically ream the anterior glenoid?
72-year-old male undergoing a shoulder arthroplasty due to rotator cuff arthropathy 65-year-old female with a glenoid retroversion of 13-degrees undergoing shoulder arthroplasty
70-year-old female with humeral anteversion of 13-degrees undergoing shoulder arthroplasty
65-year-old female with glenoid retroversion of 25-degrees undergoing shoulder arthroplasty
59-year-old male with significant glenoid bone stock deficiency and severe osteoarthritis
CORRECT ANSWER: 2
The surgeon should consider eccentrically reaming the anterior glenoid when performing a total shoulder arthroplasty on a patient with a retroverted glenoid due to posterior deficiency associated with osteoarthritic changes which is most consistent with answer choice #2.
Normal version of the glenoid is 0-3 degrees of retroversion, but when doing a total shoulder the goal should be to place the glenoid component in neutral to slight anteversion. Reaming the anterior glenoid to neutral is a technique to be considered by the operative surgeon when presented with a patient undergoing total shoulder arthroplasty with a retroverted glenoid, as failure to perform this step increases the chance for glenoid loosening. If reaming down the anterior glenoid will take away too much bone stock (down to the coracoid process), one may consider bone grafting the posterior glenoid. To perform a total shoulder arthroplasty patients will need a functioning rotator cuff and appropriate glenoid bone stock.
Clavert et al. performed cadaveric analysis to simulate glenoid retroversion of greater than 15 degrees and found that retroversion to this degree cannot be safely corrected with eccentric anterior reaming when using a glenoid component with peripheral pegs due to penetration into the glenoid vault.
Nowak et al. used 3D-CT models of patients with advanced shoulder osteoarthritis with varying degrees of glenoid retroversion and simulated glenoid resurfacing. They found that smaller size glenoid components may allow for greater version correction when using in-line pegged components, as they would be less likely to result in peg penetration.
Illustration A shows >25 degrees of glenoid retroversion seen by axial radiograph of the shoulder in a patient with advanced osteoarthritis. In this case, anterior glenoid reaming is not the correct answer and a posterior glenoid allograft reconstruction would be appropriate.
Incorrect Answers:
Answer 1: This patient should undergo a reverse total shoulder due to the lack of rotator cuff where anterior glenoid reaming is not applicable.
Answers 3: Eccentric reaming is not a useful adjunct when the humerus is anteverted
Answer 4: Excessive glenoid retroversion requires allograft reconstruction of the posterior defect instead of anterior glenoid reaming
Answer 5: Eccentric reaming is contraindicated when bone stock is insufficient to allow it.
-
A 44-year-old left-hand dominant carpenter experienced immediate left elbow pain after trying to stop a heavy object from falling two days ago. Figure A shows a clinical image of the patient upon presentation. Physical exam shows full strength with wrist flexion, wrist extension, and pronation, but notable weakness with supination of the forearm. Sensory exam shows no deficits in the forearm or hand. There is a negative milking maneuver test and a positive hook test. Radiographs are shown in Figure B. What is the next most appropriate step in management?
Sling use as needed for comfort and progressive physical therapy Allograft reconstruction of the distal biceps tendon
Ulnar collateral ligament reconstruction Distal biceps tendon avulsion repair Brachioradialis and ECRB avulsion repair
CORRECT ANSWER: 4
Distal biceps tendon avulsion repair is the most appropriate next step in management.
Distal biceps tendon ruptures occur most commonly in middle-aged men and usually involve the dominant extremity. The mechanism of injury is usually a single traumatic event with eccentric force on the flexed elbow.
Sutton et al. authored a Level 5 review of distal biceps tendon ruptures. They discuss that nonsurgical management of distal biceps tears is appropriate in the low-demand or medically ill patient. Surgical repair improves elbow flexion strength by 30% and supination strength by 40% compared to nonoperative management.
O'Driscoll et al. conducted a Level 2 study examining the accuracy of the hook test for distal biceps rupture diagnosis. They found that the hook test was abnormal in 33 of 33 (100%) patients with complete biceps avulsions, and intact in 12 of 12 (100%) with partial detachments.
Figure A is a clinical image demonstrating ecchymosis in the distal arm and antecubital fossa. Figure B shows normal elbow radiographs. Illustration A shows a normal hook test with an intact distal biceps insertion.
Incorrect Answers:
Answer 1: Non-surgical management is not indicated in a healthy, middleaged patient with a distal biceps tear of the dominant arm. Answer 2: Allograft reconstruction may be needed in chronic cases.
Answer 3 & 5: UCL reconstruction or mobile wad tendon repair is not indicated in this patient. A positive milking maneuver would be consistent with a UCL injury.
-
Early reverse total shoulder designs (before the development of the Grammont-style prosthesis) had a high failure rate due to early loosening of the glenoid component. What biomechanical feature accounted for this problem?
Glenoid component did not have a neck Humeral component too horizontal Center of rotation too lateral
Center of rotation too anterior Center of rotation too inferior
CORRECT ANSWER: 3
Early reverse ball-and-socket designs failed because their center of rotation remained lateral to the scapula, which limited motion and produced excessive torque on the glenoid component, leading to early loosening. The first modern reverse prosthesis was designed by Paul Grammont. According to Boileau et al., Grammont's design "introduced 2 major innovations (1) a large glenoid hemisphere with no neck and (2) a small humeral cup almost horizontally oriented with a nonanatomic inclination of 155 degrees, covering less than half of the glenosphere. This design medializes the center of rotation compared to earlier versions which minimizes torque on the glenoid component. Furthermore, the humerus is lowered relative to the acromion, restoring and even increasing deltoid tension. The Grammont reverse prosthesis imposes a new biomechanical environment for the deltoid muscle to act, thus allowing it to compensate for the deficient rotator cuff muscles." According to Gerber, "moving the center of rotation more medial and distal as well as implanting a large glenoid hemisphere that articulates with a humeral cup in 155 degrees of valgus are the biomechanical keys to sometimes spectacular short- to midterm results".
-
Which of the following preoperative factors is a contraindication to total shoulder arthroplasty?
Passive external rotation less than 10 degrees Eccentric posterior glenoid erosion
A 2-cm full-thickness supraspinatus tendon tear Inflammatory arthritis
A preganglionic brachial plexus injury
CORRECT ANSWER: 5
A preganglionic brachial plexus palsy, otherwise known as a root avulsion injury, presents with a flail arm and has a poor prognosis for recovery of motor function. Patients with brachial plexus palsies are not candidates for total shoulder arthroplasty due to the substantial motor and sensory deficits associated with these injuries.
In contrast, patients with a preoperative loss of passive external rotation, posterior glenoid erosion, a reparable full-thickness rotator cuff tear isolated to the supraspinatus tendon, and inflammatory arthritis are not contraindicated for a total shoulder arthroplasty.
Iannotti et al. performed a Level I prospective study in 118 patients who underwent either a total shoulder arthroplasty or a shoulder hemiarthroplasty for primary osteoarthritis. The presence of a reparable full-thickness rotator cuff tear did not adversely affect outcomes in either group but rather provided better active external rotation in the cohort receiving total shoulder arthroplasties. The authors concluded that a reparable tear of supraspinatus is not a contraindication to the use of a glenoid component.
Norris et al. compared outcomes of total shoulder arthroplasty and hemiarthroplasty performed for primary osteoarthritis in 160 patients. There were no differences in postoperative pain, function, ASES scores, or range of motion between groups for patients with reparable rotator cuff tears. The authors concluded that minor thinning and small tears of the rotator cuff can be adequately addressed at the time of surgery without adversely affecting outcomes.
Illustration A is a cervical T2 axial MRI which shows a cervical root avulsion, a form of preganglionic brachial plexus injury. Notice the perineural hyperintensity.
Incorrect Answers:
Answers 1, 2, & 3 are not contraindications to total shoulder arthroplasty and were found by Iannotti et al. to have either no effect or improved outcomes when a total shoulder arthroplasty was performed over a hemiarthroplasty. Answer 4 is an indication, not a contraindication, to a total shoulder arthroplasty when sufficient bone stock is present to support a glenoid component.
-
A 42-year-old male sustains a flail chest injury and subsequently undergoes operative stabilization of his chest wall. At first follow-up, the inferior angle of his ipsilateral scapula translates medially with any attempt at overhead activity. Injury to which of the following structures would cause this abnormality?
Spinal accessory nerve C8 and T1 nerve roots
Upper and lower subscapular nerves Thoracodorsal nerve
Long thoracic nerve
CORRECT ANSWER: 5
The clinical vignette describes medial scapular winging, which is seen after injury to the long thoracic nerve.
Medial scapular winging due to a long thoracic nerve palsy can be seen after repetitive stretching in athletes, with direct compression injury, or even iatrogenically during surgical procedures to the lateral thorax. Injury to the long thoracic nerve will eliminate the function of the serratus anterior,
which acts to protract the scapula laterally and upward and stabilize the vertebral border of scapula. This results in upper extremity weakness in forward elevation or abduction as the scapula is not stabilized against the thorax.
Meininger et al. report that lesions of the long thoracic nerve and spinal accessory nerves are the most common cause of scapular winging, although numerous underlying etiologies have been described. They report patients describe diffuse neck pain, shoulder girdle discomfort, upper back pain, and weakness with abduction and overhead activities. They also report that most cases are treated nonsurgically.
Wiater et al. review injuries to the spinal accessory nerve which causes dysfunction of the trapezius and subsequent lateral scapular winging. They note that the superficial course of the spinal accessory nerve in the posterior cervical triangle makes it susceptible to injury, and iatrogenic injury to the nerve after a surgical procedure is one of the most common causes of trapezius palsy. Most injuries are treated nonoperatively, but the Eden-Lange procedure, in which the insertions of the levator scapulae, rhomboideus minor, and rhomboideus major muscles are transferred, relieves pain, corrects deformity, and improves function in patients with irreparable injury to the spinal accessory nerve.
Illustration A shows a clinical photo of medial scapular winging, while illustration B shows a clinical photo of lateral scapular winging. Illustration C shows the long thoracic nerve during a rib fixation procedure, with the nerve sitting directly on top of the serratus anterior. The trapezius is overlying the scapula at the bottom of the photo, and the patient's head is to the right of the photo. Incorrect Answers:
Answer 1: Damage to the spinal accessory nerve would cause lateral winging. Answer 2: Damage to C8 and T1 would not cause scapular winging.
Answer 3: Damage to the upper and lower subscapular nerves would affect the teres major and subscapularis muscles.
Answer 4: Damage to the thoracodorsal nerve would affect the latissimus dorsi.
-
A patient sustains a distal biceps brachii tendon rupture. If treated non-operatively, the greatest loss of strength would be seen with which activity?
Forearm supination Forearm pronation
Elbow flexion
Shoulder forward flexion Shoulder internal rotation
CORRECT ANSWER: 1
While both elbow flexion and forearm supination strength are affected, there is a greater percentage loss of supination strength. Patients may complain of weakness and fatigue with rotational activities such as using a screwdriver. The primary elbow flexor is actually the brachialis, and therefore less weakness in flexion is reported.
Patterson reviewed distal biceps ruptures and found nonsurgical treatments had 21 55% loss of supination strength and 8 36% loss of flexion strength.
Klonz reviewed anatomic and non-anatomic repairs and found better results with anatomic repairs with 91% return of supination strength and 96% return of flexion strength. Supination strength after nonanatomic repair did not improve in 4 of 8 patients (42%-56% of the uninjured arm).
-
A 27-year-old right hand dominant construction worker falls off a scaffold onto his outstretched arm. Figure A exhibits the radiograph taken at a local emergency room.
Following treatment, he is placed in a sling and follows up at your office two weeks later. He complains of a feeling that his arm is going to 'pop out'. Which specific physical examination finding is likely to be present?
Hornblower's Test Jobe's Test
Apprehension Sign with shoulder abducted and externally rotated Speed's Test
Kim's Test
CORRECT ANSWER: 5
The patient suffered a posterior shoulder dislocation, likely injuring the posterior capsule and/or labrum. Out of all the answer choices, Kim's test assesses posterior structures. Thus, Kim's test is the physical examination finding most likely to be present.
Posterior dislocations occur less frequently than anterior dislocations, and are often missed. Following closed reduction, persistent instability can occur, usually associated with posterior capsular or labral pathology. Posteriorly directed provocative maneuvers, such as the Kim test can be positive.
Robinson et al. performed an epidemiologic analysis on 120 posterior dislocations. Recurrent instability occurred at a rate of 17.7%. Risk factors for recurrent instability included age less than 40-years-old, dislocation during seizure, and a large reverse Hill-sachs (>1.5 cm3). Kim et al. describe the Kim lesion, a separation between the posteroinferior labrum and the articular cartilage without complete detachment of the labrum, which cause persistent posterior instability.
Figure A depicts a posterior dislocation on xray. Illustration A depicts the Kim test, which is performed by having the patient seated, arm at 90° abduction, followed by flexing the shoulder to 45° forward flexion while simultaneously applying axial load on the elbow and posterior-inferior force on the upper humerus. The test is positive when there is pain. Video 1 depicts the proper way to perform a Kim Test.
Incorrect answers:
Answer 1: Hornblower's test assesses the teres minor. Answer 2: Jobe's test assesses the supraspinatus.
Answer 3: Apprehension sign assesses for anterior instability of the shoulder. Answer 4: Speed's test assesses for biceps pathology.
-
A 27-year-old male bodybuilder presents to the office with vague, deep shoulder pain and weakness with his bench press. His examination is somewhat difficult due to his large size, but no significant abnormal findings are noted. Radiographs are normal, and an MRI arthrogram is shown in Figure A. Which of the following is the most likely etiology of his complaints?
Pectoralis major rupture Supraspinatus partial thickness tear SLAP lesion
Tendonitis of the long head of the biceps Posterior labral tear
CORRECT ANSWER: 5
The clinical presentation and MRI are consistent with a Posterior labral tear.
Posterior labral tears are commonly seen in individuals that have repeated posteriorly-directed stress across their glenohumeral joint (football linemen, bodybuilders). These patients will often present with ill-described pain deep in their shoulder joint, along with decreases in shoulder strength. Focused shoulder examinations, such as the Jahnke Jerk Test or Push-pull test, can elicit pain from posterior labral tears; however, the sensitivity and specificity of these tests remain under question.
Mair et al. reviewed the outcome of posterior labral injuries in nine athletes who underwent arthroscopic repair with a bioabsorbable tack after failure of conservative management; all were
able to return to contact sports. They note that posteriorly applied forces can result in a shear-type vector that can cause posterior labral tears without capsular injury.
Bradley et al. reviewed 91 athletes with unidirectional recurrent posterior shoulder instability that were treated with an arthroscopic posterior capsulolabral reconstruction. They found that significant improvements in stability, pain, and function at a mean of 27 months postoperatively. Eightynine percent of the patients were able to return to their sport.
Figure A shows an axial MRI arthrogram of the shoulder with a posterior labral tear and an associated paralabral cyst. Illustration A is another axial shoulder MRI arthrogram cut showing a posterior labral tear (red arrow) and an associated paralabral cyst (yellow arrows).
Incorrect Answers:
Answer 1-4: The MRI does not show evidence of injury to his pectoralis major, supraspinatus tendon, superior labrum, or long head of his biceps.
-
A patient sustains a full thickness tear of their teres minor. Which of the following test/signs would most likely be positive in this patient?
Jobe's test Belly press test
Internal rotation lag sign Hornblower's sign Hawkin's sign
CORRECT ANSWER: 4
Hornblower's test is completed by asking the patient to hold their shoulder in 90 degrees of abduction and 90 degrees of external rotation. The test is positive if the arm falls into internal rotation or they are unable to actively externally rotate against resistance. This suggests teres minor pathology.
There are various tests/signs used by clinicians to detect rotator cuff pathology. The teres minor is innervated by the axillary nerve and functions to externally rotate the humerus. The hornblower's test/sign has various descriptions, but all act to determine external rotation weakness. In addition to being sensitive and specific for teres minor pathology, it can also be positive with posterior supraspinatus tears.
Walch et al. review 54 patients that underwent repair of combined supraspinatus and infraspinatus rotator-cuff tears. They found that the hornblower's sign was highly sensitive and specific for irreparable degeneration of the teres minor, while the dropping-sign was highly sensitive and specific for irreparable degeneration of the infraspinatus.
Hertel et al. prospectively review 100 patients with painful shoulders and impingement syndrome. They compared various lag signs (ERLS-external rotation lag sign, IRLS-internal rotation lag sign, drop sign) to the Jobe and lift-off signs. The ERLS was less sensitive but more specific than the
Jobe sign for the supraspinatus/infraspinatus. The drop sign was the least sensitive but was as specific as the ERLS. The IRLS was as specific but more sensitive than the lift-off sign for subscapularis tears.
Illustration A shows another variation of the hornblower's sign as originally desbribed by Arthui et
positive if the patient is unable to do this without abducting the affected arm and demonstrates the difficulty in raising the hand to the mouth in the absence of external rotation of the shoulder. The video provided shows how to perform both variations of the hornblower's test.
Incorrect Answers:
Answer 1: Jobe's test is for supraspinatus weakness and/or impingement.
Answer 2: The belly press tests positive with subscapularis pathology. Answer 3: The internal rotation lag sign is both sensitive and specific for subscapularis pathology.
Answer 5: Hawkin's sign is positive in patients with impingement.
-
Reverse total shoulder arthroplasty combined with latissimus dorsi transfer would be
most appropriate for which of the following patients?
75-year-old male with post-traumatic shoulder arthritis after a four-part proximal humerus fracture with no motor dysfunction
63-year-old male with grade 4 shoulder arthritis with severe deltoid muscle dysfunction secondary to a stroke
80-year-old female with significant rotator cuff arthropathy, a negative Hornblower sign and less than 5 degrees of external rotation lag
70-year-old female with pseudoparesis of anterior elevation and external rotation, narrowing of gleno-humeral joint and acetabularization of the acromion
82-year-old male with grade 4 shoulder arthritis and an isolated supraspinatus tear
CORRECT ANSWER: 4
Reverse total shoulder arthroplasty combined with latissimus dorsi transfer would be most appropriate in a patient with pseudoparesis of anterior elevation and external rotation, in the setting of shoulder arthritis (narrowing of glenohumeral joint and acetabularization of the acromion).
Combining a latissimus dorsi tendon transfers with reverse total shoulder arthroplasty (R-TSA) helps to restore control of active external rotation. Dysfunction with external rotation can be determined clinically with external rotation lag sign, a positive Hornblower's sign, and radiographically with fatty degeneration of the teres minor classified as stage 2 or greater according to the system of Goutallier et al. or Fuchs et al.
Gerber et al. found that R-TSA with combined lat dorsi transfer yielded minimal improvements in external rotation ROM (13 deg to 19 deg) compared to increases in shoulder ROM in flexion (94 deg to 137 deg) and abduction (87 deg to 145 deg), with this procedure.
Boileau et al. examined 17 consecutive patients treated with reverse shoulder arthroplasty and latissimus dorsi and teres major transfer (L'Episcopo). They found that external rotation increased from -21 degrees to 13 degrees (+34 degrees ). They recommend transferring both the LD and TM, rather than the LD alone as it results in better active external rotation.
Illustration A is a radiograph showing a right reverse total shoulder replacement. Illustration B shows a cadaveric image of the positioning of the latissimus dorsi tendon transfer prior to implantation of the reverse total shoulder components.
Incorrect Answers
Answer 1: Post-traumatic arthritis without motor dysfunction may be treated with total shoulder arthroplasty and there is no indication for tendon transfer. Answer 2: Severe dysfunction of the deltoid is typically considered a contraindication rather than an indication for R-TSA.
Answer 3: This patient may benefit from a R-TSA. However, there is no clinical finding of external rotation dysfunction. A positive Hornblower's sign and external rotation lag >15 degrees to the contralateral shoulder would be indicative of external rotation dysfunction.
Answer 5: Isolated supraspinatus tears are not a surgical indication for reverse total shoulder arthroplasty.
-
Figure A and B are MRI images of a 42-year-old male with symptoms of right shoulder neuropathy. If this patient has an abnormality detected on EMG and nerve conduction testing, which of the following nerves is most likely to be involved?
Subscapular nerve Axillary nerve Musculocutaneous nerve Suprascapular nerve Long thoracic nerve
CORRECT ANSWER: 4
This patient is presenting with suprascapular nerve compression secondary to a spinoglenoid cyst. Injuries of the posterior shoulder joint capsule or posterior-superior labrum can result in spinoglenoid cysts. They may lead to suprascapular nerve palsy.
Patients will present with characteristic findings of external rotation
( infraspinatus) weakness when the cyst is isolated in the spinoglenoid notch. If the cyst is located in the suprascapular notch, both external rotation weakness and abduction (supraspinatus) weakness will be present. Electromyography and MRI are the investigations of choice in depicting the etiology of this mononeuropathy.
Piatt et al. found posterosuperior labral tears in 65/73 patients who had spinoglenoid notch cysts. All patients presented with should pain and weakness. Patients undergoing surgical intervention by drainage or excision +/- arthroscopic labral repair had a better outcome than non-operative care.
Westerheide et al. reported fourteen patients who underwent arthroscopic decompression of ganglion cysts associated with suprascapular neuropathy. All patients had a labral tear intraoperatively with arthroscopic drainage and labral repair. There was not recurrence at an average of 51 months of followup.
Piasecki et al. reviewed suprascapular neuropathy. Causes include:nerve entrapment along this path, particularly at the vulnerable suprascapular and spinoglenoid notch, as well as extrinsic compression by soft-tissue masses.
Figures A is a coronal MRI showing a large hyperintense mass medial to the glenoid articulation. Figure B shows an axial MRI of the lesion posterior to the glenoid. Illustration A shows a diagram of the posterior right shoulder. The suprascapular nerve can be seen traveling through the spinoglenoid notch. Incorrect Answers:
Answers 1-3,5: Spinoglenoid cysts do not compress these nerves.
-
A 12-year-old right-hand-dominant pitcher presents with progressive right shoulder pain. He is now unable to pitch. He is tender to palpation over the lateral shoulder and has pain with rotation. An AP radiograph of the affected shoulder is shown in Figures A and a contralateral radiograph is shown in Figure B. What is the most likely diagnosis?
Septic arthritis of the shoulder SLAP tear
External impingement Internal impingement
CORRECT ANSWER: 3
proximal humerus. Patients may report a recent increase in pitching regimen. On examination, there is focal tenderness at the level of the physis. Treatment focuses on rest, physical therapy and a progressive throwing program. Pitching is often stopped for 2-3 months during rehabilitation.
Chen et al. review shoulder and elbow injuries in the young athlete. Little
Leaguer's shoulder results from epiphyseal lysis secondary to microtrauma. Pain over the anterolateral shoulder may be elicited on examination. The mainstay of treatment is 2-3 months of rest and return to pitching via a progressive throwing program.
Mcfarland et al. review techniques to prevent injuries in the throwing athletes. They note that overuse injures can be avoided when appropriate throwing mechanics are enforced and pitch counts are li
physeal widening noted especially when compared to the contralateral normal pediatric shoulder view seen in Figure B.
Incorrect Answers:
Answer 1: There is no indication of an infectious process.
Answer 2: This clinical scenario is not consistent with a SLAP tear.
Answer 4: External impingment may be a cause of subacromial bursitis and rotator cuff tears. Answer 5: Internal impingement can occur in pitchers, but is not consistent with this clinical scenario.
-
A 35-year-old carpenter has pain in the antecubital fossa that is worse with turning a screwdriver. He has undergone non-operative treatment for 6 months without relief. On physical examination his hook test is normal and there is pain and weakness with resisted supination. Radiographs are shown in Figures A-C. A MRI of the right elbow is shown in Figure D. The next most appropriate treatment is?
Exploration of the radial tunnel Superficial radial neurectomy
Detachment and repair of the biceps tendon Transfer of the biceps to the brachialis EMG with nerve conduction study
CORRECT ANSWER: 3
While complete trauamtic rupture of the distal biceps is more common, partial tears have been reported in the literature. The most common presentation is pain in the antecubital fossa worse with resisted supination.
Conservative management consists of NS
Transfer to the brachialis improves flexion strength but not supination.
Ramsey et al present a review article on distal biceps tendon injuries. They state that the most successful management of partial distal biceps tears that have failed conservative management is to surgically treat it like a complete rupture with release and surgical reattachment of the distal biceps to the radial tuberosity.
Figures A-C are normal radiographs of the elbow. Figure D is a crossreferenced axial and coronal T2 MRI that demonstrates increased signal and partial distal biceps tendon tearing. Illustration C shows the resected region of distal biceps tendon in the same patient and had an excellent functional outcome following distal biceps release and surgical reattachment with 2 double-loaded suture anchors.
Video V demonstrates The hook test for detecting complete distal biceps tendon avulsions.
-
A 49-year-old man sustains a dislocation of his left elbow that is successfully reduced and splinted. He misses his scheduled follow-up appointments and returns 6 weeks later. He is immediately enrolled in a course of vigorous physical therapy. At a repeat visit at 6 months, examination reveals that he lacks 40 degrees of elbow extension, and has flexion to 80 degrees. He is taken to the operating room for surgical release. Figures A and B are diagrams depicting the ligamentous attachments about the elbow. To restore elbow flexion, in addition to releasing the articular capsule, which ligament should be released?
Ligament A Ligament B Ligament C Ligament D Ligament E
CORRECT ANSWER: 2
In addition to capsular release, the posterior band of the medial collateral ligament (MCL) should be released.
The posterior band of the MCL is attached dorsal to the axis of rotation and has greater variation in length. It increases in length by 9 mm between 60° and 120° of flexion. Posterior band contracture leads to loss of elbow flexion. In contrast, the anterior band of the MCL (AMCL) maintains a constant length ( isometric) throughout the entire arc of movement. Anterior capsule contracture leads to loss of extension.
Wada et al. treated 14 elbows with post traumatic contracture. Through a medial incision, the ulnar nerve was freed and the posterior band and posteromedial joint capsule were excised. Mean flexion increased from 89° preop to 127° postop. Anterior capsulectomy was performed for limited extension.
Morrey et al. studied structures providing stability about the elbow. They found that the anterior capsule stabilizes the elbow to varus-valgus stress in extension, not in flexion. The anterior band of the MCL is a primary stabilizer, especially in flexion.
Figures A and B are medial and lateral illustrations of the elbow, respectively, depicting the ligamentous attachments. Illustrations A and B are radiographs are 3D CT reconstruction images of the left elbow, respectively, showing heterotopic ossification around the posterior band of the MCL.
Incorrect Answers:
Answer 1: This is the anterior band of the medial collateral ligament and should not be released. Release of the AMCL leads to valgus instability.
Answer 3: This is the transverse band of the medial collateral ligament. Release of this ligament will not improve elbow ROM.
Answer 4: This is the radial collateral ligament and should not be released. Answer 5: This is the lateral ulnar collateral ligament (LUCL) and should not be released. Release of the LUCL leads to posterolateral rotatory instability ( PLRI ).
-
A 23-year-old male sustains a dislocation of his elbow that was successfully closed reduced in the emergency room. 3 months later, the patient presents with pain and a catching sensation in his elbow. On physical exam, he is noted to have a positive lateral pivot-shift test. Incompetence of which of the following ligaments in Figure A is most commonly associated with his condition?
A B C D E
CORRECT ANSWER: 3
The patient is presenting with symptoms and physical exam consistent with posterolateral rotatory instability. Injury to the lateral ulnar collateral ligament
( LUCL), labeled C in Figure A, allows an abnormal external rotation
( supination) of the ulna on the humerus. This results in posterolateral rotatory instability. Posterolateral rotatory instability often presents as pain and recurrent clicking, snapping, clunking, or locking of the elbow. It should be noted that frank dislocations are not the most common presenting symptom. The physical exam is usually benign except for a positive lateral pivot-shift test or posterolateral rotatory drawer test. While injury to the LUCL is thought to be the primary pathology, other ligamentous stabilizers of the elbow may play a role.
Mehta et al. review posterolateral rotatory instability of the elbow. They state the instability usually results from an elbow dislocation with subsequent failure to heal of the ligamentous structures.
Patients with recurrent instability often require surgical intervention, as bracing is typically cumbersome and ineffective.
The video provided shows how to perform the lateral pivot-shift test. The patient is placed in the supine postion with forearm overhead and elbow extended. The elbow is then supinated with force and flexed to >40° while a valgus load applied. A positive result is palpable / visible clunk as the ulna and radius reduce suddenly. Illustration A shows the posterolateral rotatory drawer test.
External rotation and posterior forces are applied to the forearm attempting to sublux the radius posterior to the capitellum.
Incorrect Answers:
Answer 1: A represents the transverse bundle of the ulnar collateral ligament. Answer 2: B represents the posterior bundle of the ulnar collateral ligament. Answer 4: D represents the annular ligament.
Answer 5: E represents the anterior band of the ulnar collateral ligament.
-
Figure A is the MR image of the left shoulder of an active 47year-old painter who has been experiencing shoulder pain for 9 months. In addition to the finding shown in Figure A, MRI examination of the intra-articular portion of the biceps tendon shows fraying greater than 50%. He has not obtained relief from an 8 month course of non-operative management including non-steroidal antiinflammatory medications, physical therapy and corticosteroid injection. What is the best next step in treatment?
New course of physical therapy
Activity shutdown with 6 weeks sling immobilization
Arthroscopic superior labrum anterior to posterior (SLAP) tear repair Arthroscopic debridement and possible biceps tenotomy versus tenodesis
Arthroscopic rotator cuff repair and acromioplasty
CORRECT ANSWER: 4
This patient has a Type II SLAP lesion. These should only rarely be repaired in patients older than 40 years of age. If a source of pain refractory to nonoperative management, biceps tenotomy or tenodesis should be considered.
SLAP repair for Type II SLAP lesions is a procedure that has enjoyed a high success rate in young patients. These are generally not indicated for repair in patients greater than 40 years of age due to high rate of stiffness postoperatively. A subset of patients continue to do poorly after SLAP repair. Poor range of motion and the development of post-surgical adhesive capsulitis is often an etiology for poor results. Arthrofibrosis recalcitrant to diligent therapy over many months can be treated with arthroscopic capsular release. This is predicated on failure of a dedicated course of physical therapy as part of a non-operative management course lasting greater than six months. As the propensity for stiffness increases with age, consideration should be treated with SLAP tear debridement and biceps tenotomy or tenodesis in patients greater than 40 years old. Tenotomy or tenodesis, however, can be effective at providing pain relief in the presence of proximal biceps tendon pathology.
Katz et al. reviewed 34 patients who presented to their group for management of failed SLAP repair. 50% were Worker's Compensation cases. The mean age at the time of initial SLAP repair was 43 years. They treated these patients conservatively initially followed by revision surgery in 21 cases. All completed a course of physical therapy initially. They concluded that once a patient has failed SLAP repair, there is a high chance of further conservative treatment failing. Although revision surgery improves outcomes, 32% will continue to have a "suboptimal" result. Holloway et al. reviewed 50 patients who underwent arthroscopic capsular release for adhesive capsulitis, comparing three groups: (1) post-surgical; (2) post-fracture; and (3) idiopathic adhesive capsulitis. All patients had completed supervised physical therapy and a home exercise program for at least one year. They concluded that arthroscopic capsular release improved range of motion equally for all three groups but patients in the post-surgical group had poorer subjective pain, function and satisfaction scores.
Figure A is an MRI showing a Type II SLAP tear. Illustration A shows the classification of SLAP lesions.
Incorrect Answers:
Answer 1. The patient has failed a course of non-operative management. Answer 2. This is likely to result in marked shoulder stiffness.
Answer 3. Patients over 40 years of age are at a high risk of postoperative stiffness if undergoing SLAP repair
Answer 5. This patient does not have a rotator cuff tear.
-
A 30-year-old man undergoes arthroscopic Bankart repair for recurrent anterior dislocation. He continues to experience instability postoperatively. Examination reveals a positive apprehension test. Radiographs of both shoulders are seen in Figure A. CT scan of his left shoulder is seen in Figure B. What is the best treatment option?
Bankart repair
Humeral head bone augmentation Remplissage
Coracoid autograft Connolly procedure
CORRECT ANSWER: 4
This patient has anterior glenoid bone deficiency (inverted pear glenoid) from a large bony Bankart lesion that was not adequately addressed in the index procedure. This is best treated with bony augmentation using the Latarjet vascularized coracoid transfer.
Patients with glenoid bone defects >20-30% have a high recurrence rate
(>60%) after Bankart repair alone. Bone grafting is necessary to offer containment. Autograft options include coracoid transfer (such as the Latarjet procedure which extends the articular arc and creates a conjoined tendon sling) and iliac crest bone grafting.
Burkhart et al. addressed glenohumeral bone defects. They advise that significant bone deficits cannot be adequately addressed via arthroscopic Bankart repair alone. The Latarjet transfer creates an extra-articular platform to extend the articular arc of the glenoid.
Hantes et al. assessed Latarjet repairs using CT. They found that there is almost complete repair of a 25% to 30% glenoid defect when using the Latarjet procedure.
Figure A comprises comparison Bernageau view glenoid profile radiographs of both shoulders.
Figure B is a 3D reconstruction CT with showing glenoid bone deficiency (inverted pear deformity) with a large bony Bankart lesion. Illustration A shows the method of obtaining a Bernageau glenoid profile view. Illustration B shows the "cliff sign" of anterior glenoid bone loss.
Illustration C depicts the Latarjet procedure. Illustration D depicts reduction in the articular arc with anterior glenoid loss.
Incorrect Answers:
Answer 1: Bankart repair is not likely to be successful with a large bony Bankart lesion. Answer 2: Humeral head bone augmentation (autograft or allograft) is useful for engaging Hill Sachs lesions.
Answer 3: Remplissage is a soft tissue filling procedure (posterior capsulodesis and infraspinatus tenodesis) which also addresses engaging Hill Sachs lesions Answer 5: The Connolly procedure involves transfer of the infraspinatus and attached greater tuberosity bone block into a Hill Sachs lesion. It is analogous to the Neer-McLaughlin transfer (subscapularis and lesser tuberosity for reverse Hill Sachs lesions) in posterior instability.
-
Figure A shows an arthroscopic picture of a 62-year-old male undergoing repair of a torn subscapularis tendon. In the image shown, G represents the glenoid, H represents the humeral head, and the dotted line represents the superolateral border of the subscapularis tendon. Which two ligaments form the structure marked with the asterisk?
Inferior and middle glenohumeral ligaments Middle and superior glenohumeral ligaments Coracohumeral and coracoacromial ligaments
Coracohumeral and superior glenohumeral ligaments Superior and inferior glenohumeral ligaments
CORRECT ANSWER: 4
The coracohumeral and superior glenohumeral ligaments form a complex that marks the superolateral margin of the subscapularis tendon.
In chronic or degenerative tears, the subscapularis will often retract medially and become scarred to the deltoid fascia. This makes identification difficult during arthroscopic repair. The coracohumeral and superior glenohumeral ligaments form a complex that inserts on the superolateral margin of the subscapularis. This "comma sign" can usually be identified during arthroscopic repair making identification of the subscapularis tendon an easier task.
Burkhart and Brady present surgical pearls for arthroscopic repairs of the subscapularis. Amongst other things, they state the subscapularis is almost always repairable with proper mobilization, but an Achilles tendon allograft or a subcoracoid pectoralis major transfer may be used for a severely degenerated subscapularis.
Lo and Burkhart describe the comma sign for repair of chronic subscapularis tears. They describe how the superior glenohumeral ligament/coracohumeral ligament complex and subscapularis tendon are intimately associated, and often tear off the humerus while remaining attached to each other. This complex, when torn, forms a "comma sign," that marks the superior and lateral margins of the subscapularis tendon.
Illustration A shows why the convergence of the superior glenohumeral and coracohumeral ligaments on the superolateral border of the subscapularis is referred to as the "comma sign." Incorrect Answers:
Answers 1, 2, 3, 5: The superolateral margin of the subscapularis is attached to the coracohumeral and superior glenohumeral ligaments.
-
A 52-year-old man sustained the left elbow injury shown in Figure A while playing basketball 2.5 months ago. He underwent the procedure shown in Figure B. Post-operatively he was mobilized in a hinged brace. On examination today, his arc of elbow flexion is 75 degrees with loss of 45 degrees of full extension. His Disabilities of the Arm, Shoulder and Hand (DASH) Outcome Measure score is 45 points. What initial treatment option will likely provide the greatest improvement in this patients DASH score and functional range of motion?
Self-directed exercise therapy Supervised exercise therapy
Supervised exercise therapy with static progressive elbow splinting Continuous passive motion device
Closed manipulation under anesthesia
CORRECT ANSWER: 3
The clinical presentation is consistent with post-traumatic elbow stiffness following an elbow fracture-dislocation. Supervised exercise therapy with static elbow splinting over a 6 month period
has shown to have a significant improvement on DASH scores and functional range of motion (ROM) in patients with post-traumatic elbow stiffness.
Post-traumatic elbow stiffness is often difficult to manage. The ultimate goal of treatment is to restore a functional range of elbow motion (30° to 130°). Nonoperative modalities are considered the first-line of treatment. Aggressive physical therapy has traditionally been advocated. However, the use of static progressive elbow splinting with a turnbuckle, alongside aggressive physical therapy, has shown to provide better functional outcomes. Treatment is usually maintained over a period of 6-12 months. Surgery is considered when nonoperative therapy fails.
Doornberg et al. looked at a retrospective case series of 29 patients with posttraumatic elbow stiffness. They showed that static progressive splinting can help gain additional motion when standard exercises fail to produce additional improvements.
Lindenhovius et al. randomized sixty-six patients with post-traumatic elbow stiffness into static progressive elbow splint therapy or dynamic elbow splinting over a 12 month period. There was no significant difference in outcomes between treatment modalities. ROM increased by 40° vs. 39° at six months, respectively. DASH scores improved from 50 vs 45 at enrollment to 32 vs. 25 at six months, respectively.
Figure A shows a posterior elbow dislocation with an associated medial epicondyle fracture. Figure B shows ORIF of the fracture seen in Figure A. Illustration A shows a static progressive turnbuckle elbow splint used for posttraumatic elbow stiffness.
Incorrect Answers:
Answer 1: Self directed exercise therapy of the elbow has shown to have poor results.
Answer 2: Supervised exercise therapy is the first line of treatment for most patients with elbow stiffness. However, clinical response is variable and often unsatisfactory.
Answer 3: Continuous passive motion machines have a limited role in treating established contractures. They do not seem to improve end-range mobility in these patients.
Answer 4: Closed manipulation under anesthesia may worsen elbow stiffness and cause intraarticular damage. Manipulation causes significant swelling and inflammation with tearing of soft
tissues, causing hemarthrosis and additional fibrosis in the joint.
-
A 25-year-old right-hand baseball pitcher presents with persistent shoulder pain for the past several months in his dominant throwing arm. On physical examination, he is found to have full arc of motion with the exception of an internal rotation deficit of 30 degrees compared to his contralateral side. He is asked to complete the exercise shown in the video in Figure V. This form of rehabilitation is meant to address pathology in which anatomic structure?
Superior glenohumeral ligament Middle glenohumeral ligament
Anterior band of the inferior glenohumeral ligament Superior band of the inferior glenohumeral ligament
Posterior band of the inferior glenohumeral ligament
CORRECT ANSWER: 5
tissues in patients demonstrating symptoms of internal impingement. The sleeper stretch helps to address posterior tightness and the only posterior structure listed in the responses is the posterior band of the inferior glenohumeral ligament (posterior IGHL).
Internal impingement is a significant cause of pain in throwing athletes. It results from impingement of the articular undersurface of the posterior supraspinatus against the posteriorsuperior glenoid.
This is thought to be secondary to tightness in the posterior soft tissues including the capsule and posterior band of the IGHL. The mainstay of non-operative management is posterior capsular stretching with the sleeper stretches and cross-body adduction stretches. Heyworth et al. review the etiology, diagnosis and management of internal impingement of the shoulder. They note that repetitive contact between greater tuberosity and glenoid rim posterosuperiorly lead to impingement of the posterior rotator cuff and labrum. This occurs when the arm is externally rotated and abducted.
Tyler et al. reviewed the effects of posterior capsular stretching on alleviating symptoms in patients with internal impingement. Twenty-
Figure V is a video that demonstrates the sleeper stretch to address tightness of the posterior soft tissues. The arm is forward flexed 90 degrees and the patient lies on his side in order to stabilize the scapula while the arm is internally rotated. Illustration A depicts the sleeper stretch.
Incorrect Answers:
Answers 1, 2: Neither of these structures are implicated in internal impingement Answer 3: The anterior band of the IGHL may demonstrate increased laxity in internal impingement, not tightness
Answer 4: The superior band of the IGHL is found to have the most strain in SLAP lesions, but is
not the focus of the sleeper stretch.
-
A 28-year-old professional baseball pitcher sustains a complete rupture of his ulnar collateral ligament. He is neurovascularly intact on exam. Which of the following surgical reconstruction techniques has been shown to result in the lowest complication rate and best patient outcome?
Splitting of flexor-pronator mass, figure-of-8 graft fixation. Splitting of flexor-pronator mass, docking graft fixation.
Splitting of flexor-pronator mass, docking graft fixation, ulnar nerve transposition. Detachment of flexor-pronator mass, figure-of-8 graft fixation, ulnar nerve transposition.
Detachment of flexor-pronator mass, docking graft fixation, ulnar nerve transposition.
CORRECT ANSWER: 2
Ulnar collateral ligament (UCL) reconstruction using a flexor-pronator musclesplitting approach and a docking graft fixation technique are associated with the lowest complication rate and best patient outcomes.
Vitale et al. performed a systematic review of retrospective cohort studies evaluating UCL reconstruction techniques in overhead athletes. They demonstrated that the flexor-pronator musclesplitting approach was associated with better outcomes than detachment of the flexorpronator mass, had a lower rate of postoperative ulnar neuropathy, and a lower overal complication rate. They also found fixation of the graft utilizing the docking technique was associated with better outcomes than the figure-of-8 technique. Abandoning the obligatory ulnar nerve transposition was associated with improved patient outcomes (89% vs. 75%) and a lower rate of postoperative ulnar neuropathy (4% vs. 9%).
Rettig et al performed a case series review of 31 overhead throwing athletes with ulnar collateral ligament injuries managed nonoperatively with 3 months rest followed by rehabilitation exercises. They concluded that 42% of athletes were able to return to their previous level of competition at an average of 6 months from diagnosis (earlier than reconstruction). The authors were unable to identify any patient-specific factors (duration of symptoms, age, acuity of onset) that would predict the success of nonoperative treatment.
Illustration A shows the figure-of-8 (Jobe) graft fixation technique. It is performed by passing the tendon graft through two bone tunnels in the medial epicondyle of the humerus and through one tunnel in the ulnar sublime tubercle. The graft is then sutured to itself in a figure-of-8 configuration. Illustration B shows the docking graft fixation technique. The graft is placed in a triangular configuration through a single humeral tunnel. The suture limbs are then brought out through two separate bone holes and tied over a bony bridge on the superior aspect of the medial epicondyle.
Incorrect Answers:
Answer 1: The figure-of-8 technique is not associated with better patient outcomes when compared to the docking technique.
Answer 2: Obligatory ulnar nerve transposition during UCL reconstruction is associated with a higher rate postoperative ulnar neuropathy and worse patient outcomes, and therefore should be avoided.
Answer 3 and 5: Detachment of the flexor-pronator mass is not associated with better patient outcomes when compared to the muscle-splitting approach.
-
The right shoulder exercise seen in Figure A will put the LEAST amount of stretch on which structure?
Inferior glenohumeral ligament Coracohumeral ligament Anterior-superior capsule Superior glenohumeral ligament Posterior capsule
CORRECT ANSWER: 5
Shoulder wand exercises, as shown in Figure A, are used to increase external range of motion of the shoulder. With the arm adducted and the elbow flexed, this exercise will put the LEAST amount of stretch on the posterior capsule.
External rotation shoulder wand exercises are commonly used for the treatment of adhesive capsulitis. Adhesive capsulitis is most commonly caused by contracture of the rotator interval. The rotator interval includes the anterior-superior capsule, superior glenohumeral ligament, coracohumeral ligament and long head biceps tendon. The structure most commonly contracted is the anterior-superior capsule, which limits external rotation when the arm is adducted. Kuhn et al. showed that in the neutral position, each ligament except the posterior capsule significantly affected the torque required for external rotation. The greatest effect on resisting external rotation at 0 degrees of abduction was the entire inferior glenohumeral ligament > coracohumeral ligament
> anterior band of the inferior glenohumeral ligament > superior and middle glenohumeral ligament.
Harryman et al. looked at the role of the rotator interval capsule in passive motion and stability of the shoulder. They found operative alteration of this capsular interval was found to affect flexion, extension, external rotation, and adduction of the humerus with respect to the scapula. Limitation of external motion was increased by operative imbrication of the rotator interval and decreased by sectioning of the rotator interval capsule.
Kim et al. reviewed shoulder MRIs to determine if abnormalities of the rotator interval were correlated with chronic shoulder instability. They found a significantly larger rotator interval height, rotator interval area, and rotator interval index in patients with chronic anterior shoulder instability compared to patients without instability.
Figure A shows a patient performing an exercise to increase right shoulder external rotation with a wand/stick. The right arm is fully adducted by her side, and her elbow flexed at 90 degrees.
Incorrect Answers:
Answers 1,2,3,4: These structures are all the static stabilizers of the shoulder. Each will be subjected to a significant degree of stretch when the arm is externally rotated.
-
) A 55-year-old male returns for followup 3 months after reverse shoulder arthroplasty. He reports limited function of his right shoulder but no antecedent trauma. A radiograph of his shoulder is shown in Figure A. All of the following variables are associated with this complication EXCEPT:
History of malunited proximal humerus fracture Proximal humeral bone loss
Failed primary arthroplasty Rheumatoid arthritis
Fixed preoperative glenohumeral dislocation
CORRECT ANSWER: 4
Rheumatoid arthritis is not associated with reverse shoulder arthroplasty (RSA) dislocation. RSA dislocation is a known complication of RSA. Risks include proximal humeral bone loss, chronic fracture sequelae with malunited/ununited tuberosities, failed previous arthroplasty, and fixed glenohumeral dislocation preoperatively. An irreparable subscapularis tears may be less of an issue with newer implant designs.
Trappey et al. studied instability and infection rates after RSA. They found that the rate of instability was similar in primary and revision surgery, but the rate of infection was higher in revision surgery. Instability was highest in the fracture sequelae group because of malunited tuberosities, contractures and proximal humeral bone loss.
Favre et al. examined the effect of component positioning on RSA stability.
They found that humeral version was more important than glenoid version. Stability is improved with the humerus in neutral or slight anterversion. They recommend avoiding retroversion >10deg. Edwards et al. examined subscapularis insufficiency and the risk of RSA dislocation. They found that of 138 RSA, all 7 dislocations occurred in patients with an irreparable subscapularis.
Dislocation was also more likely in patients with complex diagnoses, including proximal humeral nonunion, fixed dislocation, and failed prior arthroplasty.
Figure A shows reverse shoulder arthroplasty dislocation. Incorrect Answers:
Answers 1, 2, 3, 5: Proximal humerus fracture malunion, proximal humeral bone loss, failed primary surgery, and fixed dislocation are all risk factors for RSA dislocation.
-
) A 45-year-old man complains of chronic right shoulder pain. He has a history of chronic steroid use because of asthma. He recently completed a course of physical therapy
and has given up his job as a laborer in favor of a desk job. Examination reveals diminished shoulder abduction strength. A radiograph of his shoulder is shown in Figure A. Which of the following surgical treatment options (Figures B through F) is the most appropriate?
Figure B Figure C Figure D Figure E Figure F
CORRECT ANSWER: 1
This patient has early stage avascular necrosis (AVN) of the humeral head without subchondral collapse/flattening, likely related to chonic steroid use.
Core decompression is indicated.
Treatment of humeral head AVN is dependent on Cruess Stage. Precollapse stages (Stage I and II) may be treated by core decompression and joint preservation. Hemiarthroplasty is used for Stage III-IV disease. Total shoulder arthroplasty is used for Stage V disease. Resurfacing may be used for Stage III disease with focal chondral defects and sufficient remaining epiphyseal bone stock for fixation.
Harreld et al. reviewed humeral head AVN. They advocate attempting core decompression and arthroscopy for Stage III disease, and then tailoring resurfacing or replacement depending on defect size.
Smith et al. reviewed 31 hemiarthroplasties for steroid-related AVN (6 Stage III, 16 Stage IV, 5 Stage V). Unsatisfactory results were found in 45%. This was associated with glenoid cartilage wear over time. However, they still believed hemiarthroplasty was appropriate for younger active patients with stage III or stage IV disease.
LaPorte et al. performed core decompression for various stages of AVN.
Results were successful in 94%, 88%, 70% and 14% of Ficat-Arlet Stages I, II, III and IV humeral head AVN respectively, and more successful for nonsteroid related cases compared with steroidrelated cases. They recommend this treatment for Stages I-III.
Figure A comprises a radiograph showing Cruess Stage II disease ("snowcap" sign indicating sclerosis, preservation of the head contour and absence of subchondral collapse, left), a T1weighted
MRI (center) and T2-weighted fat saturated MRI (right) showing a variegated pattern of osteonecrosis, but with preservation of normal head contour. Figure B shows core decompression. Figure C shows hemiarthroplasty. Figure D shows reverse shoulder arthroplasty. Figure E shows resurfacing. Figure F shows total shoulder arthroplasty. Illustration A depicts the Cruess staging system. Illustration B shows a possible algorithm for management where they use the Ficat stages adapted from the hip.
Incorrect Answers:
Answer 2: Hemiarthroplasty is an option for Stage III-IV disease (head collapse, not involving glenoid).
Answer 3: Reverse shoulder arthroplasty is not indicated in the presence of an intact rotator cuff. Answer 4: Resurfacing may be performed for focal Stage III defects, but is not appropriate for large defects. It is not appropriate if there is global humeral head involvement.
Answer 5: TSA is only indicated when the glenoid is involved (Stage V).
-
A 56-year-old otherwise healthy woman undergoes uncomplicated arthroscopic repair of a full-thickness rotator cuff tear. Prior to the procedure, the patient had attempted a long,
protracted course of physical therapy in an attempt to regain function without surgery. At her 10 day post-operative visit, she tearfully informs you that she cannot see her physical therapist because she has used up her 24-visit allotment for the year. She has 4 more weeks
until her insurance year turns over, and she does not have the financial means to pay out of pocket. What is the best next course of action?
Request a peer-to-peer review for authorization of additional visits or else the patient is likely to have a poor result
Provide a brief explanation and sheet of exercises for periscapular and cuff strengthening exercises using a resistance band
Test her range of motion and strength, and if limited, have her return weekly to your clinic for a guided home rehabilitation program
Explain to her that seeing a physical therapist during the first 6 weeks following cuff repair will not affect her range of motion one year removed from surgery
Give her a prophylactic corticosteroid injection to avoid early post-operative adhesive capsulitis
CORRECT ANSWER: 4
Early motion following rotator cuff repair has not been shown to impact range of motion and stiffness at one year post-operative clinical examinations.
Stiffness is a complication of protecting rotator cuff repairs from early re-tear with sling immobilization. However, with sling immobilization of up to 6 weeks, there is evidence of no increase in long-term stiffness. Accordingly, supervised physical therapy is not required in the first 6 weeks following arthroscopic rotator cuff repairs in order to obtain a good result.
Parsons et al. retrospectively reviewed 43 patients with full-thickness cuff tears who underwent a conservative early-postoperative protocol involving sling immobilization for 6 weeks with no formal therapy during that time. 10 /43 patient were characterized as stiff at their 6-week postoperative visit. At one year, there was no difference in the range of motion of the early stiff group compared to the non-stiff group.
Trenerry et al. collected prospective data on 209 consecutive patients undergoing primary rotator cuff repair. They found that patients in the stiffest quartile of range of motion testing at 6 weeks progressively regained range of motion by a post-operative visit at 72 weeks. The predictor of slowest recovery of early post-operative stiffness was found to be an internal rotation deficit with the patient reaching behind his or her back.
Incorrect Answers:
1: Early supervised physical therapy has not been shown to be essential to obtaining good motion post-operatively.
2 and 3: Strength testing and home strengthening programs in the early postoperative period would put the repair at risk for early failure.
5: Prophylactic corticosteroids are not indicated for prevention of postoperative stiffness, and furthermore, could theoretically interfere with healing.
-
A 62-year-old woman presents with chronic shoulder pain. On physical exam, she has anterior shoulder pain and her symptoms are reproduced with provocative testing of the biceps including supination against resistance and forward flexion of the shoulder against resistance. Internal and external rotation are painful, but her range of motion is intact. Shoulder radiograph and MRI images are shown in Figures A-E. Which of the following statements is true regarding the patient's condition?
Her clinical examination is most consistent with a SLAP tear, which should be repaired. Her biceps pathology is due to her partial tearing of her subscapularis
She has isolated degenerative biceps tendonosis and an injection may cure her symptoms
She has end-stage rotator cuff arthropathy and should consider a reverse total shoulder arthroplasty
Her subacromial impingement is causing her biceps tendon sheath to be inflamed
CORRECT ANSWER: 2
The subscapularis tendon is the most important medial restraint to subluxation or dislocation of the long head of the biceps tendon (LHBT).
Anterior shoulder pain and positive provocative biceps clinical examination tests are common in patients with concomitant rotator cuff pathology. A consequence of subscapularis tendon tears -even partial tears - is that the LHBT can subluxate medially out of the intertubercular groove, as the subscapularis tendon is the most important restraint to medial instability of the LHBT. This instability can cause both pain and inflammation around the biceps tendon, leading to pain with resisted supination (Yergason's test) or resisted forward flexion (Speeds' test).
Walch et al. wrote a case series on 71 cases of biceps tendon instability. They found that dislocated LHBTs were associated with partial or complete subscapularis tears in 96% of cases.
Maier et al. published clinical results of treating acute traumatic medial LHBT instability with open repair of the subscapularis tendon and stabilization of the LHBT. They showed equivalent functional clinical outcomes to biceps tenotomy or tenodesis, with improved cosmesis and decreased muscle cramping.
Figures A and B show Grashey and axillary lateral views of the patient and demonstrate an os acromiale. Figures C, D, and E are axillary T2 MRI images. Figure C shows the tendon of the long head of the biceps in the distal aspect of the biceps grove. Moving proximally, Figure D shows the tendon subluxated onto the lesser tubercle and Figure E shows it fully dislocated medially.
Incorrect Answers:
Answer 1: Provocative testing specific for SLAP tears is not discussed in the the questions stem. Additionally, SLAP repairs are contraindicated in this age group.
Answer 2: While she may have a component of degenerative LHBT tendonosis, the medial instability of the tendon is due to the subscapularis tear. Answer 4: Her clinical examination and imaging do not show rotator cuff arthropathy.
Answer 5: While her x-rays show a downward curved acromion, subacromial impingement does not cause instability of the LHBT.
-
A 78-year-old male presents to clinic 4 weeks after left total shoulder arthroplasty. He has not been wearing his sling and reports that he developed increased pain after slipping in the shower. He used the arm to catch himself from falling. On examination, he can flex the shoulder to 70 degrees, limited by pain. Active external rotation with arm at the side is 50 degrees and active internal rotation is 5 degrees. Passive external rotation is to 80 degrees. A radiograph of the left shoulder is shown below in Figure A. What other complaint is the patient most likely to have?
Pain with palpation of the bicipital groove Pain with palpation over the subdeltoid bursa Sensory loss over the lateral shoulder
Sensation of shoulder instability with external rotation Sensation of shoulder instability with internal rotation
CORRECT ANSWER: 4
The clinical presentation is consistent with a tear of the subscapularis, which is a well-described complication after total shoulder arthroplasty. The most likely additional complaint this patient will have is anterior shoulder instability, noticeable with external rotation of the shoulder.
Total shoulder arthroplasty is the preferred treatment for glenohumeral arthritis in patients with intact rotator cuff and good glenoid bone stock. The surgical approach involves detaching the subscapularis and capsule from the anterior humerus and dislocating the humeral head anteriorly. Post operatively, external rotation is limited to protect the subscapularis repair. If there is suspicion of a postoperative subscapularis tear, and ultrasound can be performed to confirm the diagnosis.
Miller et al. reported 7 cases of subscapularis tendon rupture after total shoulder arthroplasty, all of which were subsequently repaired. Decreased functional outcomes were observed in these patients, with lengthening techniques to address internal rotation contractures and prior surgery involving the subscapularis tendon as risk factors for rupture
Westoff et al. performed static and dynamic ultrasounds on 22 patients after total shoulder arthroplasty evaluating for numerous periarticular pathologies. The authors concluded that sonography is a useful tool for evaluation of periimplant tissues after TSA.
Figure A shows an intact left total shoulder arthroplasty without evidence of fracture, dislocation, or hardware loosening. Illustration A shows the incision for the subscapularis tendon during TSA. Incorrect Answers:
Answer 1: Pain with palpation of the bicipital groove would be expected in biceps tendinopathy Answer 2: Pain with palpation of the subdeltoid bursa would be expected in subdeltoid bursitis Answer 3: Loss of sensation over the lateral aspect of the shoulder would be expected with axillary nerve injury
Answer 5: Sensation of shoulder instability with internal rotation would be expected with posterior instability.
-
A 25-year-old lineman is referred to your office for a second opinion. 1 year ago, he underwent an arthroscopic procedure for shoulder instability. He complains of persistent sense of instability despite the surgery. Which of the following is a contraindication to revision arthroscopic labral repair for recurrent anterior glenohumeral instability?
Glenoid bone loss of 10%
Capsular attenuation from prior thermal capsulorraphy Anterior labral periosteal sleeve avulsion (ALSPA ) lesion Glenoid labral articular defect (GLAD) lesion
Combined Superior Labrum from Anterior to Posterior tear (SLAP) and recurrent Bankart lesion
CORRECT ANSWER: 2
Capsular attenuation or postthermal capsular necrosis from prior thermal capsulorraphy is a contraindicated to arthroscopic repair.
Thermal capsulorrhaphy utilizes heat generated by radiofrequency or laser ablation to cause capsular shrinkage in an effort to treat shoulder instability. However, high recurrence rates have been found, especially around two to three weeks after the index procedure, when the capsular tissue is the weakest. In the setting of recurrence following thermal capsulorrhaphy, open revision is recommended.
Creighton et al. reported on a series of 18 patients undergoing revision arthroscopic stabilization. Of the 18, 3 failed with recurrent instability, all with previous thermal capsulorrhaphy. Miniaci et al. reviewed the outcomes following thermal capsulorrhaphy noting high rates of recurrent instability, especially in the setting of initial treatment for multidirectional instability. Park et
al. reported on a series of 14 patients undergoing revision following thermal capsulorrhaphy. Ten out of 14 patients had signs of capsular thinning, insufficiency and attenuation.
Wong et al. surveyed 379 shoulder surgeons on the complications following thermal capsulorrhaphy. Capsular insufficiency and thinning were commonly associated with recurrent instability.
Hecht et al. performed thermal capsulorrhaphy and biomechanical analysis of the capsule in a sheep model. The authors found that the capsule was weakest at the 2-3 week post-operative timepoint, leading to the highest rate insufficiency, attenuation and mechanical failure at this time.
Incorrect answers:
Answer 2.Glenoid bone loss of 10% can still be addressed arthroscopically. The typical indication for open repair with a bony procedure such as Latarjet procedure is 20%.
Answers 3, 4, and 5: ALSPA lesions, GLAD lesions and combined SLAP and recurrent Bankart lesions can be addressed arthroscopically.
-
A 47-year-old landscaper presents with worsening left shoulder pain and weakness. Three years ago, he injured the left shoulder in a fall and elected for nonoperative
management to minimize time off from work. Physical therapy was effective until 6 months ago when his shoulder function worsened to the point that he is now unable to work.
Examination of his active range of motion reveals forward elevation 120° with pain, abduction 100°, IR at neutral to T8 and ER at neutral 5°. He has a positive ER lag sign and Hornblower's sign. Belly press and lift-off tests are normal. A recent radiograph is shown in Figures A. MRI images are shown in Figures B and C. Which of the following is the best treatment option?
Continue physical therapy Latissimus dorsi transfer Arthroscopic rotator cuff repair Pectoralis major transfer
Reverse total shoulder arthroplasty
CORRECT ANSWER: 2
This patient has a chronic massive posterosuperior rotator cuff tear with marked atrophy, tendon retraction and loss of external rotation strength that is impacting his daily life. The best treatment option for this middle-aged laborer with an irreparable posterosuperior rotator cuff tear is a latissimus dorsi transfer to restore external rotation strength and motion.
Irreparable rotator cuff tears are marked by: (1) Superior displacement of the humeral head (AHI < 5-7mm), (2) Fatty infiltration of the rotator cuff muscles ( Goutallier stage 3-4), (3) Increased duration of the tendon tear and (4) Profound external rotation weakness. These findings are predictive of poorquality tissue and stiffness of the muscle-tendon unit, not amenable to primary repair. In this setting, a latissimus dorsi transfer can be utilized to restore shoulder strength, function and improve pain. Relative contraindications include subscapularis deficiency, deltoid deficiency, pseudoparalysis of the shoulder and advanced glenohumeral arthritis.
Gerber et al. performed a case series analysis of 67 patients with irreparable rotator cuff tears managed with latissimus dorsi transfer. Patients with an intact subscapularis demonstrated improvement in pain, range of motion and strength postoperatively, while no improvement was noted in patients with subscapularis deficiency. The authors conclude that latissimus dorsi transfer should not be performed in the setting of poor subscapularis function.
Iannotti et al. found that better clinical results following latissimus dorsi transfer were associated with: preserved active shoulder range of motion and strength (specifically forward elevation > 90° and external rotation > 20°), synchronous firing of the transferred latissimus dorsi muscle and male gender.
Figure A is an AP radiograph of the left shoulder with superior migration of the humeral head (AHI
< 5mm) and no evidence of glenohumeral arthritis. Figures B and C show a retracted posterosuperior rotator cuff tear and Goutallier stage 4 atrophy (more fat than muscle) of the supraspinatus, infraspinatus and teres minor, rendering this tear irreparable. Illustration A shows a latissimus dorsi transfer. The latissimus dorsi tendon is positioned over the top of the humeral head, covering most of the rotator cuff defect. The tendon is then secured to the subscapularis tendon edge and lesser tuberosity anteriorly, the remnant supraspinatus and infraspinatus tendons medially, and the greater tuberosity laterally.
Incorrect Answers:
Answer 1: This patient has failed conservative management. His persistent pain, weakness and limited ROM is not likely to improve with continued physical therapy.
Answer 3: Rotator cuff repair is unlikely to be successful in this patient as discussed above. Answer 4: Pectoralis major transfer is indicated in chronic subscapularis tears. This patient has an intact subscapularis, indicated by the normal belly press and lift-off tests on exam.
Answer 5: Reverse total shoulder is reserved for elderly low-demand patients with cuff tear atrophy or massive rotator cuff tears with pseudoparalysis of the shoulder. This patient is young and active, with preserved forward elevation and no glenohumeral arthritis.
-
A collegiate swimmer develops medial winging of the scapula. If the EMG and nerve conduction studies are abnormal, the most likely nerve roots to be involved are?
C7, C8, T 1 C6, C7, C 8 C5, C6, C 7 C4, C5, C 6 C3, C4, C 5
CORRECT ANSWER: 3
Classic medial winging of the scapula is due to paralysis of the serratus anterior muscle which is supplied by the long thoracic nerve which holds the scapula to the chest wall and prevents the inferior angle of the scapula from migrating medially. It is innervated by the long thoracic nerve (C5, 6, 7).
Surgical treatment may include partial pec major transfer. Lateral winging may be caused by spinal accessory nerve palsy (CN XI, also ventral ramus C2,3,4). The nerve may be injured during neck surgery. This causes trapezius weakness, allowing the inferior pole of the scapula to migrate laterally. The modified Eden-Lange procedure may be used for this type of winging.
Gregg et al. describes paralysis of the serratus muscle in young athletes which they felt was due to repetitive traction. Full recovery usually occurs in an average of 9 months, and they recommend that surgical methods of treatment should be reserved for patients in whom function fails to return after a twoyear period.
Foo et al. describes a larger cohort of 20 patients again treated expectantly with observation and physical therapy. They reported consistent recovery but that it can take up to 2 years.
Illustration A shows a clinical photo of medial scapular winging. Illustration V is an instructional video of scapular winging. It begins with a clinical video of the condition.
-
A 62-year-old man complains of shoulder pain for 2 years. He has had 1 course of intra articular sodium hyaluronate and 6 weeks of physical therapy with little relief. Examination reveals diminished arm flexion and abduction secondary to pain. Radiographs of his shoulder are shown in Figures A and B. According to the American Academy of Orthopaedic Surgeons Clinical Practice Guidelines, what is the next best step?
Humeral head replacement arthroplasty Hemiarthroplasty and ream-and-run glenoid procedure
Cuff tear arthropathy (CTA) prosthesis
Total shoulder arthroplasty with a metal-backed cemented glenoid component Total shoulder arthroplasty with an all-polyethylene cemented glenoid component
CORRECT ANSWER: 5
This patient has end-stage glenohumeral osteoarthritis (GH OA). According to the AAOS CPG, total shoulder arthroplasty (TSA) is recommended using an allpolyethylene cemented glenoid component.
TSA is indicated for cases of end-stage GH OA. It is preferred to hemiarthroplasty. It is contraindicated in cases with insufficient glenoid bone stock (glenoid wear to the level of the coracoid), rotator cuff arthropathy or irreparable cuff tears and deltoid dysfunction. It provides good pain relief and has good survival at 10 years (>90%).
Radnay et al. performed a systematic review involving 1952 patients comparing TSA with humeral head replacement (HHR). They found that TSR provided greater pain relief, range of motion, patient satisfaction, and had lower revision rates. They recommend TSA over HHR for GH OA. Izquierdo et al. described the AAOS Clinical Practice Guidelines (CPG) regarding treatment of GH OA. This is summarized in Illustration A.
Figures A and B show end-stage GH OA with large osteophytes and subchondral sclerosis. There is significant glenoid wear and posterior subluxation (Walch B glenoid deformity). Illustration A is a table summarizing the AAOS CPG on treatment of GH OA. Illustration B shows a CTA humeral component. It is not paired with a glenoid component.
Incorrect Answers:
Answer 1: The AAOS CPG does not recommend humeral head replacement arthroplasty (resurfacing).
Answer 2: Although the AAOS CPG recommends both hemiarthroplasty and TSA as options, TSA is preferred.
Answer 3: The AAOS CPG does not recommend use of a CTA humeral component. Answer 4: The AAOS CPG does not recommend the use of metal-backed glenoid components.
Metal-backed glenoids have higher rates of revision than all-polyethylene glenoids.
-
Posterior glenohumeral dislocations are as common as anterior dislocations in which of the following patient groups?
Football players
Marfan's syndrome patients Renal failure patients Epilepsy patients
Women
CORRECT ANSWER: 4
Millett et al and Robinson et al provide review articles on posterior shoulder dislocations, which are rare clinical entities that occur during seizures and electrocution (due to tetanic muscle contraction) or as a result of high energy trauma. Robinson et al noted that poor prognostic factors associated with posterior shoulder dislocation include late diagnosis, large bony defect of humeral head, associated proximal humerus fracture, and need for arthroplasty. In Gerber's series, posterior dislocations occurred with equal frequency to anterior in a cohort of epilepsy patients.
-
What nerve is injured most commonly during the superficial dissection when repairing a distal biceps rupture through a single incision anterior approach?
Medial antebrachial cutaneous nerve Lateral antebrachial cutaneous nerve Superficial radial nerve
Ulnar nerve
Posterior interosseous nerve
CORRECT ANSWER: 2
The lateral antebrachial cutaneous nerve (LABCN) is at risk during the superficial dissection when repairing a distal biceps rupture through a single incision anterior approach.
The LABCN is the terminal cutaneous branch of the musculocutaneous nerve, which supplies sensation to the volar-lateral aspect of the forearm. The LABCN pierces the deep fascia of the arm lateral to the musculotendinous junction of the distal biceps tendon after lying on top or piercing through the brachialis muscle. It exits the arm and lies in the subcutaneous tissues of the antecubital fossa. It is important to retract this nerve laterally during the approach to the distal biceps tendon.
Cohen describes the importance of identifying the LABCN during the superficial dissection as injury to this nerve is not uncommon (5-7%). Injury to the PIN (5 %) is devastating and occurs with retractor placement during the deep dissection and the use of suspensory fixation on the posterior cortex of the radius.
The review article by Ramsey et al covers the surgical anatomy and complications of biceps surgery, including injury to the LABCN.
Illustration A shows the LABCN relative to the anterolateral approach to the elbow which is commonly used to repair distal biceps avulsions. Illustration B shows the close proximity of the LABCN to the distal biceps in a human specimen.
Incorrect Responses:
Answers 1 & 4: Should not be injured with a standard volar approach to the antecubital fossa. Answers 3 & 5: The radial nerve divides into the superficial radial and posterior interosseous nerve (PIN). These can be injured with with retractor placement, but is less common than LABCN during the superficial dissection. The PIN is particularly at risk with retractors around the neck of the radius and when passing a beath pin out the dorsal forearm such as when using a cortical suspensory fixation device.
-
An MRI of the shoulder in a patient with chronic quadrilateral space syndrome is most likely to show which of the following?
Increased intra-capsular volume Loss of intra-capsular volume Fatty atrophy of the infraspinatus Fatty atrophy of the teres minor
Fatty atrophy of the latissimus dorsi
CORRECT ANSWER: 4
Quadrilateral space syndrome involves dysfunction of the axillary nerve, perhaps by entrapment or compression, resulting in the functional denervation of the teres minor.
The quadrilateral space is a potential space formed by the long head of the triceps medially, the humerus laterally, the teres minor above, and the teres major below. The axillary nerve and posterior circumflex humeral artery travel through this space.
The Sanders article describes the MRI appearance, which is that the muscle appears streaked with white on MRI and atrophied (See illustration A) consistent with fatty atrophy. Sanders group report this finding in 3% of shoulder MRIs. The posterior circumflex humeral artery also travels with the axillary nerve as it travels through this space. Loss of capsular volume on an arthrogram study is suggestive of adhesive capsulitis.
Illustration B is a diagram which shows the borders of the quadrilateral (or quadrangular) space.
-
A 21-year-old collegiate volleyball player is noted to have weakness in external rotation and isolated atrophy of the infraspinatus on physical examination as seen in Figure A. An axial MRI image is shown in Figure B. This clinical condition is most likely caused by compression of the:
Axillary nerve at the triangular space Suprascapular nerve in the suprascapular notch Axillary nerve in the quadrangular space Suprascapular nerve in the spinoglenoid notch
Long thoracic nerve anterior to the scalenus and the first rib and posterior to the clavicle
CORRECT ANSWER: 4
The clinical presentation is consistent for a suprascapular neuropathy caused by compression of the suprascapular nerve by a cyst in the spinoglenoid notch.
The suprascapular nerve arises from the upper trunk of the brachial plexus with contributions from C5-6. It travels through the suprascapular notch of the scapula where it gives motor branches to the supraspinatus then around the spinoglenoid notch where it innervates the infraspinatus.
Compression of the nerve at the suprascapular notch will cause denervation and atrophy of both the supraspinatus and infraspinatus while compression at the spinoglenoid notch affects the infraspinatus in isolation. This is commonly seen in overhead athletes who sustain a SLAP tear and resultant spinoglenoid notch cyst as seen in the MRI. This will cause weakness and atrophy of the infraspinatus and can be noted both clinically and radiographically. Appropriate operative management is still not clear in the literature with some authors reporting a need for labral repair + cyst decompression and others reporting good outcomes with labral repair alone. Other cases have been treated with needle aspiration.
The cited reference by Cummins et al reviews the various causes, diagnosis, and treatment of suprascapular neuropathy.
The reference by Martin et al is a retrospective study of the results of nonoperative treatment of suprascapular neuropathy in which 5 had excellent results and 7 had good results.
-
Which of the following provocative tests would most likely be positive in a patient with medial epicondylitis?
Resisted forearm pronation and wrist flexion with a clenched fist Resisted forearm supination and wrist extension with a clenched fist Dynamic valgus stress test
Milking maneuver Pinch grip test
CORRECT ANSWER: 1
A provocative test for medial epicondylitis can be elicited by applying resistance to a patient with their fist clenched, wrist flexed and pronated.
Medial epicondylitis is an overuse syndrome of the flexor-pronator mass. The pronator teres (PT) and flexor carpi radialis (FCR) are thought to be most affected with this condition. It is most common in the dominant arm and occurs with activities that require repetitive wrist flexion/forearm pronation. Patients are most tender over the origin of PT and FCR at the medial epicondyle.
Resisting a patient with their fist clenched, wrist flexed and pronated can cause worsening of their pain. This maneuver can be used as a provocative test for this condition.
Cain et al. reviewed elbow injuries in throwing athletes. They comment that the common flexorpronator muscle origin provides dynamic support to valgus stress in the throwing elbow, especially during early arm acceleration and help produce wrist flexion during ball release.
Amin et al. reviewed the evaluation and management of medial epicondylitis. They report that medial epicondylitis typically occurs in the fourth through sixth decades of life, the peak working years, and equally affects men and women. Physical therapy and rehabilitation is the main aspect of recovery from medial epicondylitis, once acute symptoms have been alleviated.
Illustration A shows a video of this provocative test for medial epicondylitis. Incorrect Answers:
Answer 2: Resisted forearm supination and wrist extension with a clenched fist would be used to assess for lateral epicondylitis
Answer 3: Dynamic valgus stress test is used to asses for MCL elbow injuries Answer 4: Milking maneuver is used to asses subjective apprehension, instability, or pain at the MCL origin.
Answer 5: Pinch grip test is used to assess for anterior interosseous nerve ( AIN) injury/palsy.
-
A 72-year-old woman presents for follow-up after elbow surgery. Her radiographs are shown in Figures A and B. Which of the following pre-operative diagnoses is a relative contraindication to the use of this prosthesis design?
Acute intra-articular distal humerus fracture Malunited intra-articular distal humerus fracture Late-stage rheumatoid arthritis
Post-traumatic bony ankylosis Osteoarthritis
CORRECT ANSWER: 3
This patient has had an unconstrained total elbow arthroplasty (TEA).
Unconstrained TEA is least preferred for late-stage rheumatoid arthritis where there is significant capsuloligamentous instability and bony erosion.
Unconstrained (unlinked or resurfacing prosthesis) TEA depend on intact bony and ligamentous constraints for stability. These are appropriate for humeroulnar conditions with intact collateral ligaments and radiocapitellar articulation e.g. osteoarthritis, post-traumatic arthritis, intra-articular distal humerus fracture, and malunion of the distal humerus. Conditions with increased risk of
instability (ligamentous injury, rheumatoid arthritis) will benefit from a linked or semiconstrained prosthesis.
Mansat et al. reviewed the Coonrad-Morrey linked (semi-constrained) TEA implant in 70 patients after 5 years. They found that patients with inflammatory arthritis had higher function than those with traumatic conditions
( fractures, nonunions and post-traumatic arthritis). Survival rate was 98% and 91 % at 5 and 10 years, respectively. They concluded that this implant provided satisfactory treatment for different indications although radiolucent lines and bushing wear were a concern.
Hildebrand et al. reviewed the functional outcome of the Coonrad-Moorey prosthesis in 51 elbows after 50 months. The inflammatory arthritis group had higher performance scores than the traumatic/post-traumatic conditions group. Isometric extensor torque was found to be less than the nonoperated side. Radiolucency was noted in 11 elbows.
Figures A and B show an unconstrained TEA with radial head replacement. Illustration A shows more examples of unconstrained TEA. Illustration B shows a semiconstrained TEA. The arrow points to the anterior flange. Illustration C shows radiolucent lines around the stems. Illustration D shows severe bushing wear leading to locking mechanism failure. Illustration E is a table comparing linked and unlinked implants.
Incorrect Answers:
Answers 1, 2, 4, 5: Primary ligamentous restraints around the elbow are likely to be intact in these conditions. Both linked (semiconstrained) and unlinked ( unconstrained) prosthesis are options.
-
A professional baseball team has several pitchers with complaints of velocity loss with their pitches and shoulder pain of their dominant shoulders during spring training. Pitch counts are properly monitored. The average glenohumeral internal rotation deficit on the pitching staff is 45 degrees. The best intervention would be:
Pitchers throwing less fastballs and more changeups
Evaluate the pitchers elbows for ulnar collateral ligament acute ruptures. Increasing the weight training for the deltoid and latissimus dorsi muscles Focused stretches and therapies that address posterior capsular tightness Firing the general manager for finding pitchers that "lose their stuff"
CORRECT ANSWER: 4
Glenohumeral Internal Rotation Deficit (GIRD) is a phenomenon that occurs in baseball pitchers and is due to posterior capsular tightness. Treatment should begin with a therapy program addressing the pathologic posterior capsule.
GIRD is a phenomenon that is frequently found in high-level overhead throwing athletes, predominantly baseball pitchers. It is defined as the measured difference in internal rotation between the non-dominant arm and dominant arm. Worsening range of motion deficits are seen with increased repetitions, both over a single season and a career. GIRD > 25º is associated with development of shoulder pathologies or pain requiring periods of inactivity. Cessation of overhead throwing activities and initiation of a stretching program to address posterior capsular contractures is largely effective (90% in some series).
Burkhart et al. reviewed the conditions associated with high-level overhead throwing athletes shoulders, culminating in a theory of pathologic progression to "dead arm syndrome" (loss of velocity and effective pitching). Their theory attributes adaptive hyperexternal rotation (occurs during late-cocking / early acceleration phases of pitching) to lead to posterior-inferior capsular contracture and GIRD. Subsequent injuries to anterior structures - including SLAP lesions - would then occur.
Illustration A is a cartoon depiction of how to perform the sleeper stretch. This is a common component of a pitcher's maintenance stretching program.
Incorrect Answers:
Answer 1: While "overthrowing" pitches is associated with a multitude of pitching pathologies, pitch selection is not a described risk factor for GIRD Answer 2: Elbow UCL insufficiency does cause loss of velocity but would not account for the shoulder internal rotation deficit. Answer 3: Heavy weight training about the shoulder is associated with increased stiffness in pitchers
Answer 5: While perhaps the "right answer" for professional baseball executives, this is an obvious distractor
-
A 68-year-old man presents with severe right shoulder pain. He had a prolonged course of physical therapy and received several cortisone injections for his pain without improvement. Examination reveals pseudoparalysis of the right shoulder with a 20-degree external rotation lag with the shoulder adducted. With the shoulder placed in 90 degrees of abduction, he can actively externally rotate his shoulder. The patient was treated with a medialized reverse prosthesis shown in Figure A. Which of the following statement is true regarding this treatment option?
It is contraindicated in patients with shoulder pseudoparalysis
It can be used in patients with deltoid dysfunction when combined with latissimus dorsi transfer It shifts the center of rotation of the shoulder superior and lateral
The risk of scapular notching is increased with inferior placement of the glenoid component The risk of instability is increased with an irreparable subscapularis
CORRECT ANSWER: 5
The clinical presentation is consistent with a patient with pseudoparalysis that was treated with a reverse total shoulder arthroplasty (RTSA). The risk of postoperative instability is increased in patients with an irrepairable subscapularis when a medialized reverse prosthesis is used. Answers 1-4 are false statements.
RTSA is most commonly indicated for rotator cuff arthropathy. However, indications for use now include shoulder pseudoparalysis, anterosuperior escape of the humeral head, acute 3 or 4-part proximal humerus fractures, and greater tuberosity fracture nonunions. Contraindications to RTSA included deltoid dysfunction, insufficient glenoid bone stock, and bony deficiency of the acromion. Edwards et al. prospectively evaluated the risk of shoulder dislocation after reverse TSA. They found a significantly increased risk of dislocation (p=0.012) in patients with an irreparable subscapularis at time of surgery. There were no dislocations in the reparable group. Dislocations were more likely in patients with proximal humeral nonunions and failed prior arthroplasty. Mulieri et al. looked at the use of reverse TSA in patients with irreparable massive rotator cuff tears without evidence of glenohumeral arthritis. All outcomes were improved postoperatively, and they advocate
for reverse TSA in this subset of patients. Survivorship was over 90% at more than 4 years average follow up.
Boileau et al. evaluated the clinical outcomes of isolated biceps tenotomy/tenodesis in patients with massive rotator cuff tears and a biceps lesion. They found that the procedure can effectively treat pain and improve function in these patients. There was no difference in patients undergoing tenotomy versus tenodesis.
Figure A is a right shoulder radiograph status post RTSA with components in adequate position. Incorrect Answers:
Answer 1: Pseudoparalysis refers to loss of shoulder active motion with intact passive motion in patients due to pain. This often occurs in patients presenting with arthritis,fractures and rotator cuff tears. It is an indication for RTSA Answer 2: RTSA is contraindicated in patients with deltoid dysfunction. RTSA can be combined with latissimus dorsi transfer in patients with intact deltoid function to restore external rotation.
Answer 3: RTSA shifts the center of motion INFERIOR and MEDIALLY giving the deltoid muscle a mechanical advantage.
Answer 4: Risk of scapular notching can be DECREASED with inferior placement and adequate inferior tilt of the glenoid component.
-
Figure A shows immediate post-operative radiographs of a 75year-old patient with primary osteoarthritis. She presents 3 years later with increasing pain and weakness in the shoulder despite home physical therapy. Examination reveals limited active range of motion, with forward elevation of 80 degrees and external rotation of 50 degrees. Her deltoid function is intact. Repeat radiographs are seen in Figure B. Which treatment option would provide the best functional outcome for this patient?
Open tendon transfer
Corticosteriod injection and supervised physical therapy
Open rotator cuff repair, subacromial decompression and distal clavical excision Revision to reverse shoulder arthroplasty
Revision to cuff arthropathy hemiarthroplasty
CORRECT ANSWER: 4
This patient presents with failed total shoulder arthroplasty. The best treatment option for functional outcome would be revision to reverse shoulder arthroplasty (rTSA).
RTSA is considered a viable treatment option for patients with failed shoulder arthroplasty. It allows for improved arm elevation and abduction in the setting of nonfunctional rotator cuff muscles, as seen in this example. Despite the expanding indications for rTSA, there are high complication rates in the revision setting. Complication rates for rTSA after failed shoulder arthroplasty have been reported to be between 11-36%. This procedure should, therefore, be performed by surgeons with extensive training in reconstructive shoulder arthroplasty.
Patel et al. retrospectively reviewed 31 patients (mean age, 68.7 years) who underwent rTSA for treatment of a failed shoulder arthroplasty. They found the greatest improvement with active forward elevation from 44° preoperatively to 108 ° postoperatively (P < .001). Complications occurred in 3 patients with periprosthetic fracture.
Hattrup et al. reviewed a series of 19 patients that underwent open rotator cuff repair after shoulder arthroplasty. Out of the 19 patients only 4 shoulders were successfully repaired. They concluded that successful rotator cuff repair after shoulder arthroplasty is possible but failure is more common. Figure A shows a left total shoulder arthroplasty that is well reduced in the glenoid. Figure B shows antero-superior escape of the prosthesis, indicative of a massive rotator cuff tear.
Incorrect Answers:
Answer 1: Poor functional outcomes have been reported in patients following tendon transfers in setting of previous shoulder replacements.
Answer 2: A well-placed corticosteroid injection may improve pain it will not address the function limitations due to a nonfunctional rotator cuff.
Answer 3: Poor results have been reported in patients that underwent rotator cuff repair after shoulder arthroplasty. Subacromial decompression and distal clavicle excision would also not be recommended in this patient as this may further destabilize the shoulder.
Answer 5: Revision to a cuff arthropathy hemiarthroplasty may address the irreparable rotator cuff tear. However, best results for hemiarthroplasty have been shown in patients with marked glenoid bone loss, younger patients, or in patients with rheumatoid arthritis who maintain forward elevation of more than 90 degrees.
-
A 35-year-old man awoke following a night of heavy drinking with severe right shoulder pain and inability to raise his arm above his head. A radiograph from the emergency room is provided in Figure A. He was treated with a sling for a diagnosis of rotator cuff tear. Six weeks later, he complains of continued pain and difficulty using the arm. Which of the following is the next best step in management?
Physical therapy for range of motion followed by rotator cuff and deltoid strengthening exercises
Axillary radiograph of the shoulder
EMG to evaluate the suprascapular and axillary nerves Arthroscopic rotator cuff repair
Open subacromial decompression and latissimus dorsi transfer for massive cuff tear
CORRECT ANSWER: 2
The radiograph demonstrates overlap of the humeral head and glenoid suggesting shoulder dislocation. An Axillary radiograph is necessary to evaluate concentric reduction vs. dislocation of the shoulder. An example is provided in illustration A. Posterior shoulder dislocations can be easily be missed without the proper orthogonal views of the shoulder. Perron reviews the proper identification and emergency room care of posterior shoulder dislocation. Richardson found axillary radiographs to be more sensitive than trans-scapular radiographs for identifying posterior shoulder dislocations.
-
Recent randomized controlled trials comparing early passive range of motion to 6 weeks of immobilization after successful arthroscopic rotator cuff repair concluded that, compared to immobilization, early passive range of motion resulted in:
Higher Constant scores at 12 months
Increased rates of re-rupture as determined by ultrasound Equivalent functional outcomes
Less pain at 6 months
Inceased range of motion at 12 months
CORRECT ANSWER: 3
A series of high-quality RCTs have demonstrated that early passive range of motion has equivalent functional outcomes when compared to 6 weeks of immobilization after arthroscopic rotator cuff surgery.
Traditionally, most surgeons recommended early post-operative range of motion exercises for their patients in order to prevent adhesions and ultimately stiffness. However, recent evidence has found that there is no difference in the healing rate, range of motion or functional outcome between patients who undergo early versus delayed (i.e. initial 6 weeks of immobilization) passive range of motion exercises after arthroscopic rotator cuff repair.
Kim et al. conducted a randomized controlled trial comparing early passive range of motion vs. immobilization in 106 patients who underwent arthroscopic repair for full-thickness rotator cuff tears. They found that there was no clinically or statistically significant difference between the two groups in pain, healing or function.
Keener et al. also conducted a randomized controlled trial of 124 patients who were undergoing arthroscopic repair of a full-thickness rotator cuff tear and found no difference between early and delayed range of motion in healing and functional outcome.
Cuff & Pupello also compared early vs. delayed range of motion during the post-operative rehabilitation phase in a randomized controlled trial of 68 individuals undergoing arthroscopic rotator cuff repair and found no significant difference in range of motion or healing.
Incorrect Answers:
Answer 1, 2, 4 & 5: A number of high quality randomized trials have shown that there is no difference between early and delayed passive range of motion in healing, range of motion, pain and function.
-
A 49-year-old male presents with right shoulder pain and weakness after undergoing open cervical lymph node biopsy approximately one year ago. A pertinent finding from the physical exam is seen in Figure A, with the patients arms by his side. Physical exam finding with the arms in a position of 90 degrees of forward elevation and 10 degrees of external rotation are shown in Figure B.
What nerve is most likely injured?
Long thoracic Suprascapular Spinal accessory Axillary Thoracodorsal
CORRECT ANSWER: 3
The patient is presenting with LATERAL scapular winging which is a result of injury to the spinal accessory nerve and resultant trapezius muscle palsy.
The spinal accessory nerve is fundamental to scapulothoracic function and essential for scapulohumeral rhythm. This nerve is vulnerable along its superficial course. The majority of injuries to the spinal accessory nerve are iatrogenic and occur secondary to head and neck surgery. There is often a marked delay in recognition and initiating treatment. Surgical treatment with the Eden-Lange transfer lateralizes the levator scapulae and rhomboids (transfer from medial border to lateral border)
Camp et al. reviewed the results of 111 patients who underwent operative management of a lesion to the spinal accessory nerve. They found that the majority (~80%) of injuries were sustained iatrogenically and that diagnosis was delayed for approximately 12 months.
Pikkarainen et al. reviewed the natural history of isolated serratus palsy. They found that symptoms mostly recover in 2 years, but at least one-fourth of the patients will have long-lasting symptoms, especially pain.
Figure A depicts a patient with lateral scapular winging. Figure B demonstrates physical exam of this patient with their arms in a position of 90 degrees of forward elevation and 10 degrees of external rotation. Illustration A highlights the difference between medial and lateral scapular winging. Illustration B depicts another example of a patient with lateral scapular winging.
Incorrect Answers:
An injury to the long thoracic nerve would result in serratus anterior palsy which would lead to MEDIAL scapular winging.
An injury to the suprascapular nerve would result in weakness and wasting of the supraspinatus and/or infraspinatus.
4 . An injury to the axillary nerve would result in deltoid muscle weakness. 5 . An injury to the thoracodorsal nerve would result in latissimus dorsi weakness and would not cause scapular winging.
-
A 31-year-old right handed pitcher felt a pop in his throwing elbow during a game. He is diagnosed with a rupture to the medial ulnar collateral ligament complex of the elbow. During which phase of the overhead throwing cycle did this pitcher most likely sustain his injury? Wind-up
Early cocking Early acceleration Ball release Follow-through
CORRECT ANSWER: 3
The medial ulnar collateral ligament is subjected to the greatest tensile stress during the late cocking/early acceleration phase of throwing.
The medial ulnar collateral ligament, or medial collateral ligament of the elbow, is composed of three bundles: an anterior bundle, a posterior bundle, and a variable
transverse oblique bundle. During late cocking and early acceleration phases of the overhead throw, the medial UCL is subjected to the greatest amount of valgus stress to the elbow. During this phase, the forearm lags behind the upper arm and generates valgus stress while the elbow is primarily dependent on the anterior band of the UCL for stability. This puts the ligament at greatest risk of injury during this phase.
Fleisig et al. examined the kinetics of baseball pitching and the implications on injury mechanisms. They showed that the UCL contributes to 54% of the varus torque that is generated during the early acceleration of throwing. The position of greatest load occurred when the arm was flexed to 95 +/14 degrees with an applied valgus load.
Illustration A shows a diagram of the medial ulnar collateral ligament ligament bundles. Incorrect Answers:
Answer 1: The windup phase is benign for the elbow
Answer 2: In early cocking, the rotator cuff and deltoid are active, not the elbow.
Answer 4: Ball release is the culmination of cocking and acceleration, but the maximal joint forces have already occurred across the UCL due to the rapid combination of valgus and extension during late cocking and early acceleration.
Answer 5: In follow-through, the elbow flexors are most active to prevent hyperextension.
-
A 14-year-old elite basketball player develops acute medial elbow pain after a fall. Physical examination reveals medial elbow tenderness over the submlime tubercle, but full range of motion. The provocative tests seen in Figure A exacerbate his elbow pain. Radiographs of the elbow are normal. What would be the next best step in treatment?
Supervised elbow stretching program Therapeutic elbow arthroscopy
Static elbow external fixation for 3 to 6 weeks, then MR arthrography if pain continues Activity avoidance for 6 weeks
Serial inflammatory markers and rheumatology referral
CORRECT ANSWER: 4
Figure A shows a moving valgus stress, which is a provocative test for ulnar collateral ligament (UCL) injury and elbow valgus instability. The initial treatment would be a short period of immobilization, rest and flexor pronator strengthening in this patient population.
Adolescent UCL injuries can be effectively treated with a short period of rest and NSAIDs to control pain. As the acute inflammation resolves, the patient can be started on a supervised therapy program. This should target flexor pronator muscles, as they are important secondary dynamic stabilizers of valgus stress. Once symptoms have improved and the athlete has regained full range of motion and strength, a mediated throwing program may be initiated. Throwing athletes should be educated to avoid provocative activities during this period.
Chen et al. wrote a JAAOS article on shoulder and elbow injuries in the skeletally immature athlete. They state that surgery is reserved for older athletes with persistent valgus instability despite > 6 months of non-surgical management.
Murthi et al. reviewed recurrent elbow instability. They state the anterior bundle of the medial ulnar collateral ligament complex is the primary valgus stabilizer of the elbow. The anterior band is taut for the first 60° of elbow flexion, and the posterior band is taut from 60° to 120° of flexion. The secondary valgus stabilizers of the elbow joint include the radial head, the anterior and posterior aspects of the capsule, and the muscular forces around the joint.
Figure A is showing a moving valgus stress. Illustration A shows provocative tests for valgus instability of the elbow. The image on the left shows a valgus stress test. This assesses the anterior bundle of the medial ulnar collateral ligament complex by flexing the elbow to 25-30 degrees and applying a valgus load across the elbow. The image on the right shows milking maneuver. This assesses the posterior bundle of the medial ulnar collateral ligament complex by pulling on the
beyond 90°. Incorrect Answers:
Answer 1: This patient has full range of motion of the elbow.
Answer 2: Arthroscopy is not indicated in the acute setting, unless there is an indication for operative treatment, such as intra-articular loose body. Answer 3: While a brace would be reasonable, application of a static elbow external fixation would be too invasive for this injury. Advanced imaging may be indicated to better identify the location and severity of injuries with persistent symptoms at 6-12 weeks. These include, MRI, MR Arthrogram, ultrasound, or stress radiographs. Note, plain MRI sensitivity is 57% and specificity is 100%, while MR arthrography is 92% sensitive and 100% specific.
Answer 5: Rheumatology referral and inflammatory workup my be indicated in patients with atraumatic elbow pain with systemic involvement.
-
A young, healthy male undergoes a distal biceps repair and sustains an iatrogenic nerve injury during the procedure. Which of the following clinical findings are most likely to be seen in this circumstance? Inability to extend the thumb
Lateral volar forearm numbness Inability to flex the middle finger Medial volar forearm numbness Dorsal thumb numbness
CORRECT ANSWER: 2
The most commonly injured nerve during a distal biceps repair is the lateral antebrachial cutaneous nerve (LABCN). Injury to this nerve would result in lateral volar forearm numbness.
Distal biceps avulsions can be partial or complete. Indications for surgical management include young, healthy patients who do not wish to sacrifice function, as well as partial biceps avulsions that do not respond to conservative management. Repair of a distal biceps avulsion can be approached through either an anterior one-incision technique or a two-incision technique (BoydAnderson). The one-incision technique uses the interval between the brachioradialis (radial nerve) and pronator teres (median nerve), while the two-incision technique uses this same interval in addition to a second posterolateral elbow incision. The lateral antebrachial cutaneous nerve is the most common nerve injured during either approach.
Kelly et al. retrospectively reviewed 74 distal biceps tendon repairs, and found five sensory nerve paresthesias. The lateral antebrachial cutaneous nerve was most commonly injured, followed by the superficial radial nerve.
Cain et al. retrospectively reviewed 198 distal biceps tendon repairs, and found a 36% complication rate. Lateral antebrachial cutaneous nerve paresthesias were found in 26%, while radial sensory nerve paresthesias were found in 6%, and posterior interosseous nerve (PIN) injury in 4%.
Illustration A shows the close relationship between the lateral antebrachial cutaneous nerve (LABCN) and the distal biceps. Illustration B shows the sensory nerves of the upper extremity and their respective areas of innervation.
Incorrect Answers:
Answer 1: Inability to extend the thumb would be the result of a posterior interosseous nerve (PIN) injury.
Answer 3: Inability to flex the middle finger would be the result of a median nerve injury. Answer 4: Medial volar forearm numbness would be the result of a medial antebrachial cutaneous nerve (MABCN) injury.
Answer 5: Dorsal thumb numbness would be the result of a superficial radial nerve injury.
-
A 33-year-old female presents with left shoulder weakness. Two weeks prior to presentation, the patient experienced sudden-onset, left shoulder pain, which occurred a few days after receiving the influenza vaccine. The pain subsided over the next day, followed by gradual weakness of her shoulder and eventual general disuse of her left upper extremity. An initial visit to her primary care provider resulted in the recommendation of observation. On physical exam, there is weakness and gross atrophy of the shoulder girdle. Figures A & exhibit T2-weighted MRI images of her left shoulder. To further confirm her suspected diagnosis, she is sent for electromyography.
What is the expected result?
Normal results
Fibrillation potentials consistent with compression at the spinoglenoid notch 3 . Sharp waves and fibrillations potentials associated with the deltoid and biceps
Acute denervation of both peripheral nerve and nerve root distribution with sharp waves and fibrillation potentials
Early reinnervation with polyphasic motor unit potentials
CORRECT ANSWER: 4
This patient has Parsonage-Turner Syndrome, which, when tested on EMG during the first 3 weeks, exhibits acute denervation of both peripheral nerve and nerve root distributions with positive sharp waves and fibrillation potentials.
Parsonage-Turner Syndrome is an idiopathic disorder with an etiology that is still unknown. Typical antecedent events can involve a viral illness, recent immunization, or elective surgery. Clinical presentation is usually initiated by acute onset shoulder pain, which quickly subsides and is followed by gradual weakness. Early MRI exhibits edema in the effected muscles, and fatty infiltration in later stages. Treatment is typically non-operative, and resolution can be seen as early as 6 weeks from onset.
Tjoumakaris et al. provide a thorough review of the diagnosis and management of ParsonageTurner Syndrome. The authors report the usefulness of MRI, which exhibits early edema and later fatty infiltration in the affected muscles, and urge the use of EMG as a confirmatory diagnostic measure as well as a monitoring tool to track resolution. Early identification and diagnosis may be treated with a short course of steroids, which may help shorten symptoms.
Stutz et al. concisely summarize Parsonage-Turner Syndrome and provide typical presentation, diagnosis and management principles. The authors note the common association with viral illness and/or recent immunization along with the importance of obtaining a baseline chest radiograph to rule out a compressive Pancoast tumor. Management is typically supportive with eventual resolution.
Figures A, B, and C are T2-weighted coronal, sagittal, and axial cuts of the shoulder girdle with associated edema in the supraspinatus and infraspinatus typically seen in Parsonage-Turner Syndrome.
Incorrect answers:
Answer 1: Parsonage-Turner syndrome exhibits denervation of both affected nerve roots and peripheral nerves with positive sharp waves and fibrillations. Answer 2: These results are associated with acute compression at the spinoglenoid notch, which is not shown on MRI.
Answer 3: These results are associated with compression at the C5 nerve root.
Answer 5: These results would be seen with resolving Parsonage-Turner Syndrome, which can be seen as early as 6 weeks following initial symptom onset.
-
A total shoulder arthroplasty (TSA) would be the most appropriate treatment in which of the following arthritic patients?
1. A 75-year-old female with a longstanding history of brachial plexus palsy 2 . A 63-year-old male with a 6 month history of shoulder pain and inability to abduct past 30 degrees
A 67-year-old female with chronic shoulder pain and evidence of significant proximal migration of the humerus on x-ray
A 70-year-old female with severe shoulder pain and radiographic evidence of glenoid erosion to the coracoid process
A 72-year-old male who is 9 months status post right TKA for OA with debilitating shoulder pain and an MRI demonstrating an intact rotator cuff
CORRECT ANSWER: 5
A total shoulder arthroplasty (TSA) is indicated in the 72 year old male with debilitating shoulder pain and an intact rotator cuff on MRI. The other patient scenarios are examples of contraindications for TSA.
A TSA involves replacement of the humeral head with a metal head and resurfacing of the glenoid to a cemented all-polyethylene surface. In order to achieve optimal results, patients must be selected carefully. Patients with an irreparable rotator cuff tear, non-functioning deltoid, inadequate glenoid bone stock and brachial plexopathy are poor candidates for TSA.
Edwards et al. conducted a multicenter randomized controlled trial to compare TSA versus hemiarthroplasty in patients with primary osteoarthritis of the shoulder. They found that TSA provided better scores for pain, mobility, and activity than hemiarthroplasty at 2 year follow-up. Boileau et al. followed 45 consecutive patients who underwent reverse TSA ( rTSA) for cuff tear arthropathy (CTA), post-traumatic arthritis, and failure of revision arthroplasty. After a mean follow-up of 40 months, they found that the reverse prosthesis improved function and was able to restore active elevation in patients with incongruent cuff-deficient shoulders. They also found that
the results were less predictable and complication and revision rates were higher in patients undergoing revision surgery as compared to those patients undergoing rTSA for CTA.
Illustrations A and B show the preoperative and postoperative x-rays of a patient with characteristic OA of the glenohumeral joint that was treated with TSA.
Incorrect Answers:
Answer 1: Brachial plexus palsies lead to substantial motor and sensory deficits. As a result, TSA is contraindicated in these patients.
Answer 2: An inability to abduct past 30 degrees is suggestive of a nonfunctioning deltoid, which is a contraindication for TSA.
Answer 3: Significant proximal migration of the humerus is indicative of a rotator cuff tear, which is a contraindication for TSA.
Answer 4: Glenoid erosion to the coracoid process suggests that there is inadequate glenoid bone stock, which is a contraindication for TSA.
-
Figure A is a glenoid CT 3D reconstruction of a 26-year-old accountant who has recurrent shoulder instability. His first dislocation occurred after a fall while skiing. He has now sustained his third dislocation, which was reduced in the emergency department prior to being sent to your office. What is the most appropriate definitive treatment?
Immobilization in external rotation for 6 weeks Arthroscopic bony Bankart repair
Arthroscopic Remplissage procedure
Glenoid augmentation using coracoid transfer Glenoid augmentation using tricortical iliac crest graft
CORRECT ANSWER: 2
This patient has recurrent shoulder instability with a small bony defect of the anterior glenoid and no previous surgery. The most appropriate definitive management in this patient would be arthroscopic bony Bankart repair.
Older (>20 years old), recreational athletes with minor glenoid bone loss (<20 % of the glenoid surface area) may be treated with soft tissue stabilization procedures using suture anchors. Goals of this procedure include tightening and repairing the torn ligament and labrum to the glenoid.
Younger, contact sports athletes with large glenoid defect (>20%) may require bony augmentation type of procedures.
Lynch et al. review the clinical presentation, assessment and treatment algorithm for surgical management of bone loss associated with anterior shoulder instability. While defects larger than 25% of glenoid width should be managed with bony augmentation, they recommend soft-tissue stabilization in smaller defects.
Balg et al. analyzed 131 patients following Bankart procedure and identified following risk factors for failure: age <=20, competitive participation in contact sports, shoulder hyperlaxity, Hill-Sachs on AP radiograph, glenoid bone loss of contour on AP radiograph.
Using human cadaveric shoulders with various anterior glenoid defects sizes, The MOON Shoulder Group compared radiography, MRI and CT to determine the most reliable imaging modality for predicting bone loss. Three-dimensional CT, followed by regular CT were the most reliable and reproducible imaging modalities for predicting glenoid bone loss.
Figure A shows an en face sagittal 3D reconstruction of a glenoid with 10% surface area loss. Incorrect Answers:
Answer 1: While closed reduction and immobilization are appropriate initial management, after failing conservative management, definitive management for recurrent shoulder dislocation is surgical.
Answer 3: Remplissage procedure is indicated in setting of large Hill-Sachs lesions.
Answers 4 & 5: Glenoid augmentation procedures using coracoid transfer ( Bristow-Latarjet) and tricortical iliac crest graft would be indicated in setting of larger glenoid bone loss.
-
A latissimus dorsi tendon transfer is a well established procedure for treatment of massive irreparable posterosuperior rotator cuff tears. All of the following factors have been shown to result in worse clinical outcomes after a transfer EXCEPT?
Nonsynergistic action of the transferred muscle Fatty atrophy of the supraspinatus and infraspinatus Deficiency of the subscapularis
Absence of the coracoacromial ligament Deltoid weakness
CORRECT ANSWER: 4
A latissimus dorsi tendon transfer can be utilized in patients with a massive, irreperable rotator cuff tear involving the supraspinatus and infraspinatus. It has been reported to relieve pain and improve function in a carefully selected patient population. Those patients with deficiency of the deltoid or subscapularis, nonsynergistic muscle action after transfer, or fatty infiltration of the posterosuperior cuff have worse clinical outcomes. Absence of the CA ligament may allow anterosuperior escape in RC deficient shoulders but has not been shown to lead to worse outcomes after a tendon transfer.
The paper by Warner, et. al demonstrated that poor tendon quality, stage 3/4 muscle fatty degeneration, and detachment of the deltoid insertion each had a statistically significant effect on the Constant score noting that salvage reconstruction of a previous cuff repair had more limited gains as compared to primary. The reference by Ianotti, et. al showed that synchronous in-phase contraction of the transferred latissimus dorsi is associated with a better clinical result while improved preoperative shoulder function and general strength also positively influence the clinical result.
-
An active 68-year-old woman undergoes an uncomplicated rotator cuff repair with a double-row construct using biocomposite knotless anchors. At her two month follow up, she is noted to have increased shoulder pain, weakness and limited motion. Imaging reveals failure of the rotator cuff repair. What is the most likely mechanism of failure?
Anchor fatigue and breakage Anchor pull out from bone
Suture rupture secondary to anchor eyelet abrasion Suture pull out from the repaired tissue
Infection
CORRECT ANSWER: 4
Rotator cuff repair (RCR) failure most commonly occurs from a failure of the repaired tissue to heal with suture anchor pull out from the repaired tissue.
The overall complication rate of arthroscopic RCR is roughly 10%. Failed RCR most commonly results from failure to heal (19-94%) secondary to poor rotator cuff tissue, insufficient vascularity or poor bone quality. Other causes of RCR failure include surgical complications (deltoid disruption, infection, foreign body reaction, stiffness, neurologic injury), diagnostic errors (missed lesions of the rotator interval, long head of biceps or subscapularis tear), and technical errors (excessive tension due to lack of proper tissue mobilization, anchor pull out secondary to improper anchor placement).
George et al evaluated the causes of failed RCR and results of revision RCR. While results of revision RCR are inferior to primary RCR, arthroscopic repair yields > 60% good or excellent results. Risk factors for poor results following revision RCR include poor tissue quality, detachment of the deltoid origin and multiple previous surgeries.
Diduch et al reviewed the design and composition of various anchors used in arthroscopic shoulder surgery. Current advancements in the field include highstrength polyethylene sutures, new biocompatible anchor materials (PEEK, biocomposite) and modified designs including knotless systems. With improved strength of the current anchors and repair constructs, the most common mode of arthroscopic RCR failure is now related to tissue failure occurring at the tissue-anchor interface.
Cole et al discussed the different primary rotator cuff repair constructs, including single row, double row, transosseous and transosseous equivalent. The authors concluded that construct selection depends on tear acuity, size and tissue quality. For acute tears < 12mm in anteroposterior length, singlerow configuration likely has sufficient strength to maintain the repair and promote healing. For more chronic tears, poor tissue quality, or tears > 1215 mm in the anteroposterior dimension, the authors recommend double-row or transosseous-equivalent repair to better restore the anatomic footprint and provide optimal mechanical stability to achieve healing. Illustration A is an algorithm from George et al detailing the decision-making process when considering revision RCR for a symptomatic failed RCR.
Incorrect Responses
Answer 1: Anchor breakage is a less common cause of RCR failure.
Biocomposite anchors exhibit high load-to-failure and result in fatigue failure less commonly than metal anchors.
Answer 2: Anchor pull out from bone is a less common cause of RCR failure and is associated with poor bone mineral density.
Answer 3: Suture rupture secondary to abrasion against the anchor eyelet is a much less common cause of RCR failure and occurs more often with metal anchors, which have relatively sharper and rougher edges as compared to biocomposite anchors.
Answer 5: Infection is an uncommon complication of RCR (~1.9%).
-
A 32-year-old cross-training athlete awakens with severe left neck and shoulder pain after a day of intense upper body training. Aside from a recent viral illness, he is otherwise healthy. His pain improves, but two weeks later he notes significant left shoulder weakness. Examination reveals weakness of shoulder abduction, forward elevation and external rotation with the arm at his side. Radiographs are normal. Electromyography demonstrates 2+ positive sharp waves and fibrillations. Sensory nerve conduction studies show reduced amplitudes. MRI of the brain, cervical spine and shoulder are shown in Figures A-D, respectively. Which of the following is true of his prognosis?
Decompression will result in improved muscle strength and function.
The patient can expect a gradual return of muscle strength without long term functional deficits. Immunomodulators may decrease the number and severity of his relapses. 4 . Arthroscopic repair will result in the best functional outcomes given the patient's high activity level.
5 . Prognosis is poor, with progressive loss of muscle strength and ultimately respiratory failure.
CORRECT ANSWER: 2
The patient has Parsonage Turner syndrome (PTS), a self-limiting disorder with spontaneous resolution of neuropathic pain, followed by gradual return of muscle strength and function.
Parsonage Turner syndrome (neuralgic amyotrophy, brachial neuritis) is an uncommon disorder characterized by severe shoulder pain and subsequent muscle paralysis, atrophy and sensory loss of the shoulder girdle as pain resolves. The most common associated risk factor is viral illness (2555%). EMG can be useful to confirm the diagnosis and will show positive sharp waves and fibrillations consistent with acute denervation. There is no defined treatment protocol for PTS. Early corticosteroid use has been postulated to improve pain and accelerate recovery. Treatment modalities include physical therapy to alleviate traction on involved nerves, as well as NSAIDs and opiates for pain control. Though patients can experience residual muscle weakness for months to years, recovery is usually complete.
Yanny et al. reviewed the use of MR imaging to diagnose denervation syndromes of the shoulder, including suprascapular nerve entrapment, quadrilateral space syndrome, and Parsonage Turner syndrome. In early PTS, STIR or T2-weighted sequences will demonstrate diffusely increased signal intensity in one or more muscles innervated by the brachial plexus.
Sumner et al. reviewed the history of idiopathic brachial neuritis. The nerves most frequently involved include: suprascapular, long thoracic, axillary and musculocutaneous. The author proposed that the etiology of PTS may be multifactorial, with combined inflammatory and mechanical interactions. One hypothesis that exists suggests that the mobility of the upper trunk allows for weakening of the blood-nerve barrier that normally prevents soluble immune factors or cells from coming into contact with the peripheral nervous system.
Figures A and B are normal MRI studies of the brain and cervical spine. Figures C and D are T2weighted MRI studies that show diffuse high-signal intensity involving the supraspinatus and infraspinatus muscles.
Illustration A is a table from Tjoumakaris et al depicting risk factors associated with development of PTS. Illustration B is a table from Tjoumakaris et al highlighting the differential diagnosis for PTS.
Incorrect Responses:
Answer 1 - Describes the prognosis of a suprascapular notch cyst, which can also result in denervation of the supraspinatus and infraspinatus muscles. A cyst was not visualized on the MRI. Answer 3 - Describes the prognosis of multiple sclerosis. MRI of the brain did not reveal focal demyelination diagnostic of MS.
Answer 4 - Describes the prognosis of a rotator cuff tear. MRI demonstrated an intact rotator cuff. Answer 5 - Describes the prognosis of amyotrophic lateral sclerosis (ALS). ALS is a pure degenerative motor disorder and does not result in sensory abnormalities (reduced sensory nerve conduction amplitudes), as seen in this patient.
-
A 78-year-old woman falls down the stairs and sustains the fracture shown in Figure A. An intraoperative photograph is shown in Figure B. The fracture is deemed not reconstructible with open reduction and internal fixation. In this patient, which of the following is a contraindication to total elbow arthroplasty?
Excessive mediolateral compression because of failure to recognize central comminution
Ulnar nerve transposition
Failure to recognize radial head impaction Olecranon osteotomy
Failure to recognize retrocapitellar comminution
CORRECT ANSWER: 4
Olecranon osteotomy is a contraindication to total elbow arthroplasty (TEA) so as to maintain competency of the elbow extensor mechanism.
Chevron intraarticular olecranon osteotomies provide excellent exposure for intra-articular fractures and low transcondylar fractures. First, part of the anconeus and the flexor carpi ulnaris are released off the olecranon to identify the position of the osteotomy, which is performed at the apex of the semilunar notch, with preservation of at least 1 cm of intact olecranon proximal to the apex of the osteotomy. The olecranon osteotomy is made near to completion with an oscillating saw and completed with a sharp osteotome cracking through the articular cartilage. The osteotomized olecranon is then reflected superiorly with the triceps tendon.
Githens et al. performed a systematic review comparing TEA and ORIF for fracture. They state that the advantages of TEA include faster rehabilitation, earlier motion and improved short-term outcomes. Disadvantages include weight-bearing restriction, component wear and loosening, and potential for revision arthroplasty. They found no clinically evident difference in functional outcomes of patients (ROM and functional scores). Total complications were more frequent after TEA but major complications are more frequent after ORIF.
Figure A shows a comminuted intraarticular distal humerus fracture. Figure B shows exposure of the fracture through an olecranon osteotomy. The ulnar nerve is identified with a blue vessel loop. Incorrect Answers:
Answer 1: Central comminution is not a contraindication to TEA.
Answer 2: Ulnar nerve transposition is not a contraindication to TEA. Answers 3 and 5: The radiocapitellar joint is secondarily important in performing a TEA.
-
Which of the following labels in Figure A corresponds to the ligament that acts as a static restraint of the shoulder against translation anteriorly at 45 degrees of abduction and external rotation?
A B C D E
CORRECT ANSWER: 4
The middle glenohumeral ligament (MGHL), which is labelled D in Figure A, acts as a static restraint against anterior translation at 45 degrees of abduction and external rotation.
The glenohumeral ligaments are thickened bands of the joint capsule that extend from the inferior and anterior glenoid and labrum, to the anatomic neck of the humerus. The superior glenohumeral ligament (SGHL) is an inferior stabilizer of the adducted shoulder. The anterior band of the inferior glenohumeral ligament (IGHL) is a primary restraint against anterior-inferior dislocation at 90 degrees of abduction and external rotation. The posterior band of the IGHL acts as a static restraint against posterior-inferior translation in internal rotation and adduction.
Burkart et al. reviewed the anatomy of the glenohumeral ligaments and found them to be complex and variable, with their function highly dependent on the position of the humerus with respect to the glenoid.
O'Connell et al. used cadaveric specimens to investigate the ligamentous stabilizing mechanisms preventing anterior instability in the glenohumeral joint. They found that as the shoulder is brought from 45° to 90° of abduction and externally rotated, more strain is placed on the IGHL. Thus, the authors suggested that the integrity of the anterosuperior band of the IGHL should be addressed at the time of surgical repair for anterior instability of the shoulder.
Figure A depicts the fibrous capsule (A); long head of biceps (B); SGHL (C); MGHL (D); IGHL
(E). Illustration A shows the appropriately labelled anatomic structures from Figure A. Incorrect Answers:
Answer 1: This is the fibrous capsule Answer 2: This is the long head of biceps
Answer 3: This is the superior glenohumeral ligament Answer 5: This is the inferior glenohumeral ligament
-
Reverse total shoulder arthroplasty with a latissimus dorsi transfer would be the most appropriate treatment for which of the following patients?
Previous shoulder arthrodesis and complete brachial plexus injury
Humeral head avascular necrosis with partial thickness infraspinatus tear 3 . Failed hemiarthroplasty with the inability to perform active external rotation with the arm abducted 4 . Primary shoulder osteoarthritis with 10 degree of glenoid retroversion 5 . Primary rotator cuff arthropathy with active forward shoulder flexion >100 degrees and external rotation >50 degrees CORRECT ANSWER: 3
Reverse total shoulder arthroplasty with a latissimus dorsi transfer would be most appropriate in a patient with failed shoulder hemiarthroplasty and the inability to perform active external rotation with the arm abducted
R-TSA has become the mainstay treatment for rotator cuff arthropathy. In the presence of severe loss of active elevation and external rotation, combined latisimus dorsi transfer and reverse total shoulder arthroplasty can restore elevation and external rotation, respectively. This may be used in the primary or revision setting.
Frankle et al. report the results of sixty patients with rotator cuff deficiency and glenohumeral arthritis who were followed for a minimum of two years. All were treated with R-TSA. Their study
showed that forward flexion increased from 55 to 105 degrees, and abduction increased from 41 to 102 degrees.
Boileau et al. followed 45 patients with severe cuff tear arthropathy and advanced atrophy/fatty infiltration of the infraspinatus or teres minor muscles. All patients were treated with R-TSA and a modified L'Episcopo procedure ( latissimus dorsi and teres major transfer). Mean active elevation increased from 74 degrees preoperatively to 149 postoperatively, and external rotation increased from -21 to 13 degrees.
Illustrations A and B show the classic findings of rotator cuff arthropathy. There is significant acromial acetabularization and femoralization of humeral head. Other features include: asymmetric superior glenoid wear, osteopenia, "snowcap sign" due to subchondral sclerosis and anterosuperior escape. Illustration C shows a left shoulder after conversion from hemiarthroplasty to reverse total shoulder arthropathy.
Incorrect Answers:
Answer 1: Complete brachial plexus injury and shoulder arthrodesis are contraindications for reverse total shoulder arthroplasty.
Answer 2: Humeral head AVN with partial thickness infraspinatus tear would most likely be treated early with conservative treatments (e.g. physiotherapy, stretching, etc). Surgical treatment would most likely involve humeral head resurfacing.
Answer 4: Primary osteoarthritis of the shoulder can be treated with total shoulder arthroplasty. Reaming the anterior glenoid to neutral is a technique to be considered by the operative surgeon when presented with a patient undergoing total shoulder arthroplasty with a retroverted glenoid. Answer 5: Primary rotator cuff arthropathy with active forward shoulder flexion >100 degrees and external rotation >50 degrees can be treated primarily with a reverse total shoulder arthroplasty or hemiarthroplasty with a cuff tear arthropathy head. Active external rotation indicates functioning posterior rotator cuff muscles.
-
A 24-year-old collegiate football lineman presents with posterior shoulder pain for 6 months which failed to improve with a course of physical therapy. Examination reveals full range of shoulder motion with mild clicking. Based on findings on his MR arthrogram, arthroscopic examination is performed in the lateral position with the camera in the posterosuperior portal. The findings are seen in the video. Which of the following preoperative physical examination findings is most likely to be positive with this lesion?
Pain in the bicipital groove on supination against resistance, with the elbow flexed to 90° and forearm pronated
Painful clunk following a downward and posterior-directed force applied to the proximal arm, with the arm elevated 125° upward and internally rotated.
Pain with downward pressure with shoulder abduction to 90°, forward flexed to 30° and internally rotated
Weakness with pressing the abdomen with palm of the hand with the shoulder in internal rotation
Weakness to external rotation with the arm held 90° abducted
CORRECT ANSWER: 2
This patient has a posteroinferior capsulo-labral tear. The Kim test descibed in #2 is most likely to be positive. To perform this test, a downward and posterior-directed force applied to the proximal arm, with the arm elevated 125 ° upward and internally rotated (Illustration C). Pain indicates a positive test and may be accompanied by a clunk.
Posterior labral tears are a common cause of pain in football offensive linemen who block with their arms forward flexed 90 degs and the humeral head is pushed out posteriorly. Dislocations occur in only 2-5% of patients with posterior labral tears. Traumatic cases arise from falls or collisions with the arm forward flexed, adducted and internally rotated. In posterior instability, posterior labral tears are located at the posteroinferior quadrant of the glenoid (6- to 9-position for the right shoulder and the 3- to 6-
(2005) reviewed a series of 19 arthroscopic and 12 open posterior shoulder stabilization surgeries for posterior instability. They found better outcome scores with the arthroscopic suture anchor repair and capsular plication group than the open suture anchor repair and capsulorrhaphy group. They recommend surgical treatment for these lesions, but state that the most favorable outcomes were in the arthroscopic group.
Kim et al. presented a series of 15 patients with a Kim lesion (concealed avulsion of the posteroinferior labrum) treated with labroplasty and capsular shift. They found that these lesions were characterized by an intact superficial labral attachment and concealed detachment of the deep labrum. The outcome was satisfactory in all by 1 patient, and stable in all unidirectional instability patients. They recommend surgeons convert this concealed incomplete lesion to a complete tear and
repair it with the posterior band of the inferior glenohumeral ligament to prevent persistent posterior instability.
Kim et al. compared the Kim test with the Jerk test for detection of posteroinferior labral lesions. The jerk test produces sharp pain when the humeral head compresses and glides over the torn posterior labrum. The Kim test is a modification of the jerk test where downward force displaces the humeral head more inferiorly. The sensitivity of the Kim test was 80%, specificity was 94%. The accuracy of the jerk test was: sensitivity, 73%; specificity, 98%. The Kim test was more sensitive in detecting a predominantly inferior labral lesion. The jerk test was more sensitive in detecting a predominantly posterior labral lesion. The sensitivity in detecting a posteroinferior labral lesion increased to 97% when the 2 tests were combined.
The video demonstrates a probing of a posterior capsulo-labral tear, as seen through a posterior viewing portal. Illustration A demonstrates a Kim lesion with an intact superficial labrum. Once incised, detachment of the deep labrum from the glenoid rim is visible. Illustration B is an MR arthrogram showing a Kim lesion. There is incomplete avulsion/cystic lesion but an intact junction between the glenoid articular cartilage and posterior labrum. Illustration C shows the Kim test.
Illustration D shows the Jerk test. Incorrect Answers
Answer 1: The Yergason sign is suggestive of biceps tendinitis. Answer 3: The Jobe's test examines for supraspinatus weakness.
Answer 4: The belly press test is suggestive of subscapularis weakness. The patient may attempt to cheat by drawing the elbow posterior to the body plane, and flexing the wrist.
Answer 5: Weakness of the teres minor suggests axillary nerve pathology.